You are on page 1of 87

NOTAS DE AULA

RESOLUO DE PROBLEMAS

Benedito Tadeu Vasconcelos Freire


NOTAS DE AULAS - SEDIS. UFRN

WWW. SEDIS . UFRN . BR

Notas de Aulas, materila complementar para os alunos do Curso de Licenciatura em Matemtica Distncia,
da Universidade Federal do Rio Grande do Norte.

Primeira Verso, janeiro de 2016


Contedo

1 Introduo . . . . . . . . . . . . . . . . . . . . . . . . . . . . . . . . . . . . . . . . . . . . . . . . . . . . 5

2 ESTRATGIAS . . . . . . . . . . . . . . . . . . . . . . . . . . . . . . . . . . . . . . . . . . . . . . . . . . 9
2.1 Entender, antes de fazer! 9
2.2 A busca do plano 21
2.3 Procure semelhanas com outros problemas 33
2.4 Comear pelo fcil torna fcil o difcil 37
2.5 Experimente, procure algo que seja invariante 40
2.6 Faa um desenho e, dependendo da situao, pinte s cores. 43
2.7 Modifique o enunciado, para ver se lhe ocorre um caminho possvel 50
2.8 Explore a simetria 53
2.9 Reduo ao Absurdo. 56
2.10 Suponha o problema resolvido (ou olhe de trs para diante) 60
2.11 O Princpio da Induo. 62

3 Problemas Diversos . . . . . . . . . . . . . . . . . . . . . . . . . . . . . . . . . . . . . . . . . . . 67

4 ABREVIATURAS . . . . . . . . . . . . . . . . . . . . . . . . . . . . . . . . . . . . . . . . . . . . . . . . 85
4.1 Abreviaturas 85

5 Bibliografia . . . . . . . . . . . . . . . . . . . . . . . . . . . . . . . . . . . . . . . . . . . . . . . . . . . 87
1. Introduo

Escrevemos estas Notas de Aula com o intuito de que possam ser utilizadas pelos estudantes do
Curso de Licenciatura em Matemtica Distncia, da Universidade Federal do Rio Grande do Norte, como
texto complementar para o uso na disciplina Resoluo de Problemas.

Na formao do professor de matemtica deve prevalecer o ponto de vista de que, na sua prtica docente
futura, ele exera um ensino que priorize a aprendizagem por compreenso. Logo, ele tem de ter ao longo
de seu curso um treinamento que o leve as atitudes positivas, facilitando o desenvolvimento do gosto e do
prazer das descobertas. Entendemos que a disciplina Resoluo de Problemas rene as condies para este
treinamento, criando um ambiente que torne o aluno um ativo participante do processo de aprendizagem.

Da prtica da nossa convivncia com os estudantes, verificamos que a maioria deles no apresenta desen-
voltura na prtica da resoluo de problemas. Estas notas pretendem mostrar algumas ideias que podero
apontar ao estudante caminhos facilitadores no sentido de tornar a atividade de resoluo de problemas
menos rida.

Para resolver problemas de forma clara, o estudante tem que comunicar suas ideias de maneira coerente,
com organizao e com argumentos estruturados, permitindo a ele prprio e aos outros o entendimento
fcil de seus argumentos e a interpretao perfeita de suas ideias. A Matemtica a linguagem apropriada
para a resoluo de problemas nas diversas reas do conhecimento, por ser concisa, ter muitos recursos e
no permitir ambiguidades. Por isso, uma linguagem da Cincia. Como um reconhecimento por suas
qualidades indispensveis formao do cidado, a Matemtica a nica disciplina que estudada em todos
os pases do mundo e em todos os nveis educacionais.

Galileu Galilei 1 foi quem introduziu a matemtica como linguagem da cincia.


1 GALILEU GALILEI Galileu Galilei (1564 -1642 ), italiano, foi um fsico, matemtico, astrnomo e filsofo. Galileu Galilei foi

personalidade fundamental na revoluo cientfica. Foi o mais velho dos sete filhos do msico italiano, alaudista, Vincenzo Galilei e de
Giulia Ammannati. Viveu a maior parte de sua vida em Pisa e em Florena, na poca integrantes do Gro-Ducado da Toscana.
Galileu Galilei desenvolveu os primeiros estudos sistemticos do movimento uniformemente acelerado e do movimento do pndulo.
Descobriu a lei dos corpos, enunciou o princpio da inrcia e o conceito de referencial inercial, ideias precursoras da mecnica
newtoniana. Galileu melhorou significativamente o telescpio refrator e com ele descobriu as manchas solares, as montanhas da Lua,
as fases de Vnus, quatro dos satlites de Jpiter, os anis de Saturno, as estrelas da Via Lctea. Estas descobertas contriburam
decisivamente na defesa do heliocentrismo. Contudo a principal contribuio de Galileu foi para o mtodo cientfico, pois a cincia
assentava numa metodologia aristotlica.
O fsico desenvolveu ainda vrios instrumentos como a balana hidrosttica, um tipo de compasso geomtrico que permitia medir
6 Captulo 1. Introduo

De acordo com Paul Halmos 2 , o corao da matemtica so seus prprios problemas. "A maior parte de
cada vida significativa passada com a soluo de problemas; uma parte considervel da vida profissional de
tcnicos, engenheiros, cientistas, etc., vivida na busca de soluo de problemas de matemtica. dever de
todos os professores e dos professores de matemtica em particular, expor seus alunos para problemas muito
mais do que aos fatos", disse ele.

O uso da Matemtica como linguagem exige um conhecimento mnimo para poder ser utilizada. Por
isso, o estudante necessita de situaes que permitam exercitar essa linguagem, e uma delas so os empregos
dos mtodos de resoluo de problema.

Uma referncia no estudo da arte de resolver problema so os trabalhos do professor Polya. 3

comum uma diferenciao entre problema e exerccio, veja, por exemplo [ZEITZ], pgina 1. O
exerccio uma questo que testa o domnio do estudante numa tcnica que est sendo focada ou que foi
recentemente coberta. Exerccios podem ser difceis ou fceis, mas, de uma maneira geral, eles nunca so
intrigantes, e normalmente fica claro como proceder no sentido de como encontrar a soluo. Por outro lado,
um problema uma questo que no pode ser respondida imediatamente. Problemas so muitas vezes
abertos, paradoxais, s vezes indecifrveis e exigem investigao antes que se pode chegar a uma soluo.

Para oportunizar a resoluo de problemas, apresentamos no texto problemas de Olimpada de Matem-


tica de vrios pases, por serem intrigantes, criativos e desafiadores. No texto, identificamos a origem da
quase totalidade dos problemas aqui apresentados. Agradecemos a todos aqueles que queiram nos apresentar
a origem dos problemas ainda no identificados, para que possam constar numa prxima edio destas Notas
de Aula.
ngulos e reas, o termmetro de Galileu e o precursor do relgio de pndulo. O mtodo emprico, defendido por Galileu, constitui
um corte com o mtodo aristotlico mais abstrato utilizado nessa poca, devido a isto Galileu considerado como o "pai da cincia
moderna". Fonte: htt p : //pt.wikipedia.org/wiki/GalileuG alilei. Acessado em 17/03/2014.
2 PAUL HALMOS - (1916 - 2006) foi um matemtico estadunidense nascido na Hungria, que fez avanos fundamentais nas reas

de Lgica Matemtica, Teoria da Probabilidade, Estatstica, Teoria dos Operadores, Teoria Ergdica e Anlise Funcional (em especial,
nos espaos de Hilbert ). Ele tambm foi reconhecido como um grande expositor da matemtica.
Halmos chegou em os EUA aos 13 anos de idade, mas nunca perdeu seu sotaque hngaro. Halmos obteve seu Bacharelado na
Universidade de Illinois, graduando em filosofia e especializando-se em matemtica. Ele levou apenas trs anos para obter o grau, e
tinha apenas 19 anos quando se formou. Comeou um doutorado (Ph.D.) na filosofia, mas, passou para a matemtica, graduando-se em
1938 sob a orientao do Professor Joseph L. Doob, com a dissertao intitulada: A Invariantes de Certas Transformaes Estocsticos:
A Teoria Matemtica de Sistemas de Jogos.
Pouco depois de sua formatura, Halmos foi para o Instituto de Estudos Avanados. Seis meses depois, ele estava trabalhando com John
von Neumann, que se revelou uma experincia decisiva. Na sua permanncia no Instituto de Estudos Avanados, Halmos escreveu seu
primeiro livro, Espaos Vetoriais de Dimenses Infinitas, que imediatamente estabeleceu sua reputao como um excelente expositor da
matemtica.
Halmos ensinou na Universidade de Syracuse , na Universidade de Chicago (1946-1960), na Universidade de Michigan, na Universidade
da Califrnia, em Santa Barbara (1976-1978), na Universidade do Hava, e na Universidade de Indiana, onde se aposentou. Desde
sua aposentadoria, em 1985, at sua morte, ele era ligado ao departamento de Matemtica da Universidade de Santa Clara. Fonte:
htt p : //en.wikipedia.org/wiki/PaulH almos. Acessado em 17/03/2014
3 GEORG POLYA - George Plya (1887 1985) hngaro, professor de matemtica de 1914 a 1940 no Instituto Federal de

Tecnologia de Zurique, na Sua, de 1940 a 1953 na Stanford University, nos Estados Unidos. Plya permaneceu como Professor
Emrito da Stanford o resto de sua vida e carreira. Ele trabalhou em uma variedade de tpicos matemticos, incluindo sries, teoria dos
nmeros, anlise matemtica, geometria, lgebra, combinatria e probabilidade.
No incio de sua carreira, Plya escreveu com Gbor Szeg, matemtico hngaro famoso, dois influentes livros de problemas: Problemas
e Teoremas em Anlise (I: Srie, Clculo Integral, Teoria das Funes e II, Teoria das Funes, Zeros de polinmios, Determinantes,
Teoria dos Nmeros ,Geometria.....). Mais tarde, ele se dedicou ao estudo da Heurstica, um mtodo ou processo criado com o objetivo
de encontrar solues para um problema, para identificar mtodos sistemticos de resoluo de problemas a uma maior descoberta e
inveno em matemtica para os estudantes, professores e pesquisadores. Ele escreveu cinco livros sobre o assunto: How to Solve it,
traduzido para o portugus como A Arte de Resolver Problema, Matemtica e Raciocnio Plausvel (Volume I: Induo e Analogia em
Matemtica, e Volume II: Padres de Plausvel Inference) e descoberta matemtica: Na compreenso, aprendizado e ensino Problem
Solving (volumes 1 e 2).
Em How to Solve It, Plya fornece sugestes heursticas para resolver uma gama de problemas, incluindo os problemas matemticos e
os no-matemticos. O livro inclui conselhos para ensinar os alunos de matemtica e uma mini-enciclopdia de termos heursticos. Foi
traduzido para vrias lnguas e j vendeu mais de um milho de cpias. O fsico russo Zhores I. Alfyorov, (Prmio Nobel em 2000),
elogiou, lembrando que ele era um f. O livro ainda usado em educao matemtica.
Alm de suas obras que abordam diretamente a resoluo de problemas, Plya escreveu outro livro chamado Mtodos Matemticos em
Cincias, com base em um trabalho de 1963 apoiado pela National Science Foundation, editado por Leon Bowden, e publicado pela
Associao Matemtica da Amrica (MAA), em 1977. FONTE: htt p : //en.wikipedia.org/wiki/GeorgeP olya.
7

Esperamos que os estudantes possam apreciar as ideias que permitem a resoluo dos problemas aqui
apresentados.

Todos os erros e equvocos so de nossa responsabilidade. Sero bem vindos comentrios apontando
eventuais erros, como tambm alternativas para melhorar o texto.

Natal, janeiro de 2016

Benedito Tadeu Vasconcelos Freire


benedito@gmail.com
2. ESTRATGIAS

J clssica e bem conhecida, a formulao que fez Polya das quatro etapas essenciais para a resoluo de
um problema, que constituem o ponto de partida de todos os estudos posteriores:
Entender, antes de fazer
Traar um plano para resolv-lo
Colocar o plano em prtica
Comprovar os resultados
No que se segue, vamos desenvolver estas etapas, exercitando a resoluo de problemas.

2.1 Entender, antes de fazer!


Quando algum lhe prope um problema, um jogo, um quebra-cabea, inicialmente, assegure-se que enten-
deu a fundo os dados do problema, as regras do jogo e o possvel lugar que tem cada uma dessas informaes,
como elas se encaixam umas com as outras.

Resumindo: para resolver um problema, imprescindvel que voc conhea o que dado e, exatamente,
o que o problema pede.

Diante de um problema, faa a si mesmo as seguintes perguntas:


Qual a incgnita?
Quais so as quantidades dadas?
Quais so as condies dadas?
O que o problema pede?

A soluo de um problema consiste em ligar, por passos lgicos, os dados do problema ao que nele se
pede.

Exemplo-1:

Duas velas de comprimentos iguais e de distintas espessuras comeam a queimar ao mesmo tempo.
Uma delas vai queimar completamente em 4 horas, a outra em 5 horas. Durante quantas horas as
velas queimaro at que o comprimento de uma delas seja 3 vezes o comprimento da outra?

Soluo
10 Captulo 2. ESTRATGIAS

Depois de ler o problema (se preciso for, mais de uma vez, talvez at em voz alta), de forma a saber
exatamente quais as quantidades dadas, quais as condies dadas (hipteses) e o que se pede, vamos traar
um plano para a soluo.

Vamos chamar de d o comprimento das duas velas, de A a vela que queima completamente em 4 horas e
de B a vela que queima completamente em 5 horas.
Assim, a razo pela qual elas queimam dada por:
d d
Vela A : e Vela B : .
4 5
Depois de t horas queimando, o comprimento C de cada vela ser dado por:
 t  t
CA = d 1 CB = d 1 .
4 5
Queremos saber o tempo no qual o comprimento da vela B ser 3 o comprimento da vela A. Assim,
temos que:
 t  t  t  t 5t 4t 
d 1 = 3d 1 1 = 3 1 = 3
5 4 5 4 5 4
40
4(5 t) = 15(4 t) 20 4t = 60 15t t = .
11
40
Portanto, as velas queimaro durante 11 horas para que o comprimento de uma delas seja 3 vezes o
comprimento da outra.

Exemplo-2:

Dois barcos partem ao mesmo tempo das margens opostas de um rio e viajam com velocidades
constantes, mas diferentes. Eles passam um pelo outro quando esto a 700 metros de uma das
margens e cada um continua para a outra margem do rio, de onde eles retornam. Na sua viagem de
regresso os barcos passa novamente um pelo outro desta vez a uma distncia de 400 metros da
margem oposta. Qual largura do rio?

Soluo

(Problema proposto por W.C. Rufus, na revista American Mathematical Monthly, 47, fevereiro de 1947, veja
[BRIGGS], pag. 9.

Depois de ler o problema (se preciso for, mais de uma vez, talvez at em voz alta), de forma a saber
exatamente quais as quantidades dadas, quais as condies dadas (hipteses) e o que se pede, vamos traar
um plano para a soluo.
Aqui, facilita se fizermos um desenho ilustrativo da viajem dos dois barcos com o movimento de ida e vinda
atravs do rio.

barco 2

barco 1
400 m 700 m
2.1 Entender, antes de fazer! 11

Para a soluo do problema, um fato importante o que afirma que as velocidades dos barcos so constantes
mas diferentes.
Quando trabalhamos com velocidade constante a relao que estabelece a distncia percorrida

distncia = (velocidade) (tempo gasto)

Agora, com o desenho acima em mente, devemos atentar para um fato que a chave da soluo: o tempo
gasto por cada um dos barcos para acontecer o primeiro encontro entre eles o mesmo que o tempo gasto
pelo outro. Alm disso, dado a localizao deste encontro: 700 metros de uma das margens.
Observe que o tempo gasto pelos dois barcos para acontecer o segundo encontro, a 400 metros da margem
oposta, tambm o mesmo para os dois barcos.
Assim, com estas duas observaes, podemos escrever as duas relaes entre as variveis do problema: d a
largura do rio, que o que queremos encontrar, v1 , e v2 as velocidades dos barcos 1 e 2, respectivamente.
O barco 1, viajando com velocidade v1 , no primeiro encontro, percorreu d 700 metros, enquanto o
barco 2, viajando com velocidade v2 , no primeiro encontro percorreu 700 metros. Logo, para o primeiro
encontro, teremos para o barco 1:

d 700 700
tempo gasto = = . ()
v1 v2
Para o barco 2, temos:
700 + d 400 d 700 + 400 300 + d d 300
tempo gasto = = tempo gasto = = . ()
v1 v2 v1 v2

Podemos rescrever a equao () como:

v1 d 700
= . ( )
v2 700

De maneira anloga, a equao () pode ser reescrita como:

v1 300 + d
= . ( )
v2 d 300

Fazendo ( ) = ( ), obtemos

d 700 300 + d
= d 2 1700d = 0 d (d 1700) = 0 d = 1700, pois d 6= 0.
700 d 300
Portanto, a largura do rio 1700 metros.

Exemplo-3:

No meu carro, uma determinada marca de pneu dura 40.000 km em uma roda dianteira ou 60.000
km em uma roda traseira. Ao longo da viagem, trocando os pneus dianteiros pelos traseiros, qual a
maior distncia que consigo percorrer com um conjunto de quatro destes pneus?

Soluo

Depois de ler o problema (se preciso for, mais de uma vez, talvez at em voz alta), de forma a saber
exatamente quais as quantidades dadas, quais as condies dadas (hipteses) e o que se pede, vamos traar
um plano para a soluo.

A chave da soluo conhecer o desgaste de cada pneu dianteiro e traseiro em cada 1 quilmetro rodado.
Como cada pneus dura 40.000 km em uma roda dianteira, segue que o desgaste por cada 1 quilmetro
rodado igual a
1
40.000
12 Captulo 2. ESTRATGIAS

Como cada pneus dura 60.000 km em uma roda traseira, segue que o desgaste por cada 1 quilmetro
rodado igual a
1
60.000
Logo, a mdia de desgaste dos 4 pneus dada por
1 1 1 1 1  1 2 2 
+ + + = + =
4 40.000 40.000 60.000 60.000 4 40.000 60.000
1 1 1  1 3+2  1
= + = = .
4 20.000 30.000 4 60.000 48.000
Portanto, considerando que eu otimizo o uso dos pneus, a distncia mxima que consigo deles rodar 48.000
quilmetros. Isso ser alcanado atravs do rodizio dos pneus dianteiros com os traseiros aps 24.000 km.

Exemplo-4:

Uma empresa vende mensalmente 400 unidades de um produto a R$ 120,00 cada, proporcionando
um faturamento bruto de R$ 48.000,00. Observou-se que para cada real descontado no preo eram
vendidos 20 unidades a mais por ms. Calcule o desconto que proporciona o maior faturamento
possvel e qual esse faturamento.

Soluo
Depois de ler o problema (se preciso for, mais de uma vez, talvez at em voz alta), de forma a saber
exatamente quais as quantidades dadas, quais as condies dadas (hipteses) e o que se pede, vamos traar
um plano para a soluo.
Observe que o faturamento varia como o desconto dado ao produto. Vamos chamar de x o desconto que
maximiza o faturamento, que vai ser a incgnita que queremos encontrar.
O preo do produto com o desconto vai ser 120 x.
Para cada unidade do desconto so vendidos 20 produtos a mais, alm dos 400 que j eram comercializados.
Logo, sero vendidos 400 + 20x produtos.
Portanto, o faturamento pode ser expresso pela funo:
F(x) = (400 + 20x) (120 x) = 20x2 + 2000x + 48000
Agora, observe que a expresso F(x) est definida para todo nmero real x. No entanto, os valores de x
adequados ao problema so aqueles satisfazendo 0 x < 120, caso contrrio, teramos ou um preo do
produto maior do que os R$ 120,00, ou teramos um preo negativo. A expresso de F(x) pode ser reescrita
como:
F(x) = 20x2 + 2000x + 48000 = 20(x2 100x 2400) = 20(x2 100x + 2500 2500 2400) =
= 20[(x 50)2 ] + 20 4900 = 20(x 50)2 + 98000.
Agora, basta observar que o valor de F(x) atinge um mximo quando x 50 = 0. Ou seja, F(x) atinge
um mximo quando x = 50.
Portanto, o desconto que proporciona o maior faturamento possvel R$ 50,00 e o faturamento mximo
igual a F(50) = 20(50 50)2 + 98000 = 98.000 reais.

Exemplo-5:

Um barril contm 64 litros de vinho. Substitumos 16 litros de vinho por 16 litros de gua.
Suponhamos que o vinho e a gua se misturam uniformemente e que o volume da mistura seja a
soma dos dois volumes. Em seguida, substitumos 16 litros da mistura por 16 litros de gua.
Esperamos que se misturem e voltamos a fazer mais uma vez mais esse procedimento. Quantos
litros de vinho (misturados com a gua) permanecem no barril?

Soluo
2.1 Entender, antes de fazer! 13

(Canguru Matemtico 2005) Depois de ler o problema (se preciso for, mais de uma vez, talvez at em voz
alta), de forma a saber exatamente quais as quantidades dadas, quais as condies dadas (hipteses) e o que
se pede, vamos traar um plano para a soluo.

No total, fizemos trs operaes de substituir 16 litros por 16 litros de gua. Vejamos, com a ajuda da
tabela a seguir, quantos litros de vinho permanece no barril logo aps a primeira operao.
gua Vinho Total
Inicio 0 64 64
Retira-se 16 litros de Vinho - Operao 1 -16 -16
Acrescenta-se 16 litros de gua +16 +16
Resultado Final da Primeira Operao 16 48 64

fcil ver que, logo aps a primeira operao, a proporo entre a quantidade de gua e a quantidade de
vinho de 1 para 3. Portanto, esta proporo se mantm quando se retira 16 litro da mistura para fazer a
substituio por 16 litros de gua. Isto significa dizer que, ao se retirar 16 litros da mistura, retiramos 4
litros de gua e 12 litros de vinho.
Na tabela a seguir veremos quantos litros de vinho permanece no barril logo as a segunda operao:
gua Vinho Total
O que resta aps a primeira operao 16 48 64
Retirando-se 16 litros da mistura - Operao 2 -4 -12 -16
Acrescenta-se 16 litros de gua +16 +16
Resultado Final Aps a Segunda Operao 28 36 64
Assim, de maneira anloga, fcil ver que, aps a segunda operao, a propro entre gua e vinho de
7 para 9. Portanto, quando retiramos 16 litros do barril para efetuarmos a terceira operao, retira-se 7
litros de gua e 9 litros de vinho. Finalmente, na tabela a seguir veremos quantos litros de vinho permanece
no barril logo aps a terceira operao:
gua Vinho Total
O que resta aps a segunda operao 28 36 64
Retirando-se 16 litros da mistura - Operao 3 -7 -9 -16
Acrescenta-se 16 litros de gua +16 +16
Resultado Final Aps a Segunda Operao 37 27 64
Portanto, permanece no barril 27 litros de vinho.
Exemplo-6:

Necessita-se programar uma dieta com dois alimentos S e T . Cada unidade do alimento S contm
100 calorias e 15 gramas de protenas. A unidade do alimento T contm 200 calorias e 15
gramas de protenas. A dieta requer um mnimo de 1.000 calorias e 90 gramas de protenas. Se o
preo de cada unidade do alimento S 400 reais e de cada unidade do alimento T de 300
reais, quantas unidades de cada alimento deve conter a dieta para minimizar o custo?

Soluo

(Olimpada de Matemtica do Per) Depois de ler o problema (se preciso for, mais de uma vez, talvez at em
voz alta), de forma a saber exatamente quais as quantidades dadas, quais as condies dadas (hipteses) e o
que se pede, vamos traar um plano para a soluo.

Suponha que a dieta contm s unidades do alimento S e t unidades do alimento T . Assim, a quantidade
de calorias da dieta seria igual a
100s + 200t,
que, pelas hipteses do problema essa quantidade deve ser no mnimo 1.000. Ou seja,

100s + 200t 1000 s + 2t 10. (1)


14 Captulo 2. ESTRATGIAS

A quantidade de protena da dieta seria igual a

15s + 10t,

que, pelas hipteses do problema essa quantidade deve ser no mnimo 90. Ou seja,

15s + 10t 90 3s + 2t 18. (2)

Por outro lado, o custo da dieta dado por

400s + 300t (3).

Assim, queremos minimizar a expresso (3) usando as restries (1) e (2), tendo sempre em conta que
s 0 e t 0, pois se tratam de quantidades.
A idia da soluo multiplicar (1) por uma certa quantidade M e a equao (2) por um certo nmero N
para obter a expresso 400s + 300t.
Fazendo isso, obtemos as equaes:

M + 3N = 400 e 2M + 2N = 300,

que admite como soluo M = 25 e N = 125. Logo, multiplicamos a equao (1) por 25 e a equao
(2) por 125, e somamos:
400s + 300t 250 + 2250 = 2500,

e para garantir que o valor mnimo de 400s + 300t seja 2500, devemos ter:

s + 2t = 10 e 3s + 2t = 18,

que admite como soluo s = 4 e t = 3.


Portanto, o menor custo da dieta se consegue com s = 4 e t = 3.

Exemplo-7:

Uma caixa contm p bolas brancas e q bolas pretas. Alm dessas bolas, existe uma pilha de
bolas pretas. Retiram-se duas bolas da caixa. Se elas so da mesma cor, uma bola preta da pilha
colocada na caixa. Se elas so de cores distintas, a bola branca colocada de volta na caixa. Este
procedimento repetido at que o ltimo par de bolas seja removido e que uma ltima bola reste na
caixa. Qual a probabilidade de que esta ltima bola seja branca ?

Soluo

(Olimpada de Matemtica da Austrlia-1983) Depois de ler o problema (se preciso for, mais de uma vez,
talvez at em voz alta), de forma a saber exatamente quais as quantidades dadas, quais as condies dadas
(hipteses) e o que se pede, vamos traar um plano para a soluo.

Basta observar que, quando retiramos duas bolas brancas, o nmero de bolas brancas diminui de 2. Se
retiramos duas bolas de cores distintas, o nmero de bolas brancas no se altera. Isto significa que a paridade
do nmero de bolas brancas no muda ao longo de todas as retiradas de pares de bolas (Se inicialmente par,
continua par. Se inicialmente for mpar, continua mpar).
Portanto, se p um nmero par, a ltima bola no pode ser branca -a probabilidade zero!
Se p um nmero mpar, a ltima bola tem de ser branca -a probabilidade 1!
2.1 Entender, antes de fazer! 15

Exemplo-8:

Dado o tabuleiro 7 7, a seguir:

6
4

pede-se para completar com nmeros as casas vazias, de maneira tal que a soma dos nmeros em
trs casas consecutivas (na mesma linha ou na mesma coluna) seja sempre 20.
Quando completamos com nmeros as casas vazias, que nmero ocupa o lugar de x?

Soluo

(Olimpada de Matemtica do Per) Depois de ler o problema (se preciso for, mais de uma vez, talvez at em
voz alta), de forma a saber exatamente quais as quantidades dadas, quais as condies dadas (hipteses) e o
que se pede, vamos traar um plano para a soluo.

Observe que o fato de que a soma dos nmeros em trs casas consecutivas seja sempre 20, significa dizer
que o primeiro e o quarto desses nmeros so iguais:

a b c d d

pois, temos que:


a d = (a + b + c) (b + c + d) = 20 20 = 0 = a = d.
Assim, na sexta coluna podemos escrever o nmero 6 nas casas (6, 1) e (6, 4) e podemos escrever o
nmero 5 nas casas (4, 4) e (4, 1), veja figura a seguir:

6
4

5 5 6

5 x 6

Portanto, pela hipteses do problema, temos:

5 + x + 6 = 20 = x = 9.
16 Captulo 2. ESTRATGIAS

Exemplo-9:

Pinta-se as casas de um tabuleiro retangular alternadamente branca e preta. Em seguida, escreve-se


em cada casa do tabuleiro um nmero inteiro. Sabe-se que a soma de todos os nmeros escritos nas
casas de uma linha e a soma dos nmeros escritos nas casas de uma coluna um nmero par.
Prove que a soma de todos os nmeros escritos nas casas pretas um nmero par.

Soluo

(Olimpada de Matemtica da Ucrnia-1997) Depois de ler o problema (se preciso for, mais de uma vez,
talvez at em voz alta), de forma a saber exatamente quais as quantidades dadas, quais as condies dadas
(hipteses) e o que se pede, vamos traar um plano para a soluo.

Vamos supor que a casa superior esquerda esteja pintada de branco, veja figura a seguir, para o caso de
um tabuleiro 8 5.

Como a soma de todos os nmeros escritos nas casas de uma linha qualquer (ou coluna qualquer) par,
podemos concluir que a soma de todos os nmeros escritos no tabuleiro par. Assim, para provar o que
queremos, basta provar que a soma dos nmeros escritas nas casas brancas seja um nmero par.
Agora, observe que, a soma dos nmeros escritos na primeira linha (contada de cima para baixo), mais
a soma dos nmeros escritos na terceira linha, na quinta linha etc. mais a soma dos nmeros escritos na
primeira coluna (contada da esquerda para direita), mais a soma dos nmeros escritos na terceira coluna, na
quinta coluna etc. igual soma de todos os nmeros escritos nas casas pretas mais duas vezes a soma dos
nmeros escritos em algumas casas brancas. Como, por hiptese, essa soma par, segue que a soma dos
nmeros escritos nas casas pretas par, como queramos provar.

Exemplo-10:

Em torno de um crculo, escrevem-se 2016 zeros e um 1. A nica operao permitida escolher


um nmero e mudar seus dois vizinhos, de 0 para 1 ou de 1 para 0. Aplicando a operao sucessi-
vas vezes, possvel mudar todos os nmeros para 1? E se comessemos com 2015 zeros e um 1?

Soluo

(Adptada da Olimpada de Matemtica Rioplatense-1997) Depois de ler o problema (se preciso for, mais de
uma vez, talvez at em voz alta), de forma a saber exatamente quais as quantidades dadas, quais as
condies dadas (hipteses) e o que se pede, vamos traar um plano para a soluo.

possvel com 2016 zeros, mas no possvel para 2015 zeros.


Observe que cada vez que se aplica a operao permitida, a paridade da soma dos nmeros no muda.
Tendo 2016 zeros, agrupamos os zeros em 504 grupos de 4, e escolhemos o primeiro e o terceiro zero de
cada um desses grupos para aplicar a operao.
No caso de se ter 2015 zeros e um 1, impossvel comear com uma soma par e terminar com uma soma
mpar no final, que quando todos os nmeros sejam iguais a 1.
2.1 Entender, antes de fazer! 17

Exemplo-11:

Na figura a seguir, temos um quadrado de lado medindo 10 cm. Calcular a rea da regio hachurada,
sabendo-se que os pontos A, B, C e D so os pontos mdios dos lados do quadrado dado.

D



A C



B

Soluo

Depois de ler o problema (se preciso for, mais de uma vez, talvez at em voz alta), de forma a saber
exatamente quais as quantidades dadas, quais as condies dadas (hipteses) e o que se pede, vamos traar
um plano para a soluo.

A idia juntar duas cpias da figura dada e chamar as regies congruentes em branco de a e b, veja a
figura seguir.


b
a
b a b

a
b
a a
b

Agora, observe que juntando as regies a e b, obtemos uma regio de rea igual regio hachurada.
Com isso, fcil ver o quadrado original fica dividido em 5 quadrados menores, todos congruentes ao
quadrado hachurado. Isto significa dizer que a rea da regio hachurada um quinto da rea do quadrado
dado. Portanto, a rea hachurada igual a:

102 100
= = 20 cm2 .
5 5

Exemplo-12:

Escreve-se no quadro negro uma fileira com n sinais menos (-). Dois jogadores, A e B, disputam
o jogo seguinte, em que jogam alternadamente. O jogador A comea. Uma jogada consiste em:
ou substituir um dos sinais menos por um sinal mais (+) ou dois sinais menos adjacentes por dois
sinais mais . Aquele jogador que no puder fazer seu movimento perde. Imaginando que ambos os
jogadores fazem suas melhores jogadas, o jogador A possui uma estratgia vencedora?

Soluo

(ANDREESCU-SAVCHEV, pgina 1) Depois de ler o problema (se preciso for, mais de uma vez, talvez at
em voz alta), de forma a saber exatamente quais as quantidades dadas, quais as condies dadas (hipteses)
e o que se pede, vamos traar um plano para a soluo.
O jogador A sempre vence. A estratgia do jogador consiste em:
18 Captulo 2. ESTRATGIAS

Se n par.
Em seu primeiro movimento, o jogador A muda para mais cada um dos dois sinais menos localizados
no centro da fileira. Ento, depois dessa mudana, sempre que o jogador B fizer uma jogada, o
jogador A muda para mais o(s) sinal (ais) que estejam nas posies simtrica, em relao ao centro
da fileira, das aqueles mudados por B. Com isso, o jogador A far a ltima jogada possvel.
Se n mpar.
Em seu primeiro movimento, o jogador A muda para mais o sinal menos localizados no centro da
fileira. Ento, depois dessa mudana, sempre que o jogador B fizer uma jogada, o jogador A muda
para mais o(s) sinal (ais) que estejam nas posies simtrica, em relao ao centro da fileira, das
aqueles mudados por B. Com isso, o jogador A far a ltima jogada possvel.
Exemplo-13:

Seja x um nmero real no nulo. Sabendo-se que x + 1x = 3, encontre x6 + x16 .

Soluo
Depois de ler o problema (se preciso for, mais de uma vez, talvez at em voz alta), de forma a saber
exatamente quais as quantidades dadas, quais as condies dadas (hipteses) e o que se pede, vamos traar
um plano para a soluo.
Temos que
1  1 2 1 1
x + = 3 = x + = 32 x2 + 2 + 2 = 9 x2 + 2 = 7. ()
x x x x
Agora, de (), temos que
 1 3 1 1 1 1  1
x2 + 2 = 73 x6 + 6 + 3x4 2 + 3x2 4 = 343 x6 + 6 + 3 x2 + 2 = 343.
x x x x x x
Portanto, substiutindo () na ltima expresso, temos
1 1
x6 + 6
+ 3 7 = 343 x6 + 6 = 343 21 = 322.
x x
Exemplo-14:

De trs pessoas, A, B e C, sabe-se que algumas delas sempre falam a verdade e algumas delas
sempre mentem. As pessoas fazem as seguintes afirmaes:
A diz que B e C sempre mentem.
B nega que seja um mentiroso.
C diz que B um mentiroso.
Quantas dessas pessoas mentem e quantas falam a verdade?

Soluo
Depois de ler o problema (se preciso for, mais de uma vez, talvez at em voz alta), de forma a saber
exatamente quais as quantidades dadas, quais as condies dadas (hipteses) e o que se pede, vamos traar
um plano para a soluo.
A melhor maneira de resolver o problema examinar todas as possibilidades, a partir das trs afirmaes
feitas. Vamos introduzir a notao: V para quem fala a verdade e M para quem mente.
Agora vamos fazer uma tabela com todas as possibilidades, a partir das trs afirmaes feitas.
A B C
1 Todos mentem M M M
2 Todos falam a verdade V V V
3 Dois falam a verdade V V M
4 Dois falam a verdade V M V
5 Dois falam a verdade M V V
6 Dois mentem M M V
7 Dois mentem M V M
8 Dois mentem V M M
2.1 Entender, antes de fazer! 19

Como vimos acima, a partir das trs afirmaes, temos 8 possibilidades.


Suponha que a possibilidade 1, todos mentem, seja verdadeira. Assim, a pessoa A mente quando diz que
B e C mentem. Isto significa dizer que, pelo menos uma delas fala a verdade. Logo, a possibilidade todos
mentem falsa.
Suponha que a possibilidade 2, todos falam a verdade, seja verdadeira. Assim, a pessoa C mente quando
diz que B um mentiroso. Isto significa dizer que, pelo menos uma pessoa mente. Logo, a possibilidade
todos mentem falsa.
De modo anlogo, fcil ver que as possibilidades 3, 4, 5 e 7 so falsas.
Assim, as possibilidades reais de acontecer so as possibilidades 6 e 8. Portanto, existem duas pessoa que
mentem e uma que fala a verdade.

Exemplo-15:

Bolas de tnis de mesmo raio r so hermeticamente embaladas em latas cilndricas com 3 bolas,
onde elas apenas tocam a lata na superfcie lateral, no topo e no fundo, veja figura a seguir.

d 3d

Qual maior: a altura da lata embalagem ou a medida da circunferncia de cada bola?

Soluo
Depois de ler o problema (se preciso for, mais de uma vez, talvez at em voz alta), de forma a saber
exatamente quais as quantidades dadas, quais as condies dadas (hipteses) e o que se pede, vamos traar
um plano para a soluo.
A resposta : a medida da circunferncia de cada bola.
Observe que a altura da lata igual a 3d, onde d o dimetro da circunferncia de cada bola. A medida da
circunferncia de cada bola igual 2r = 2 d2 = d > 3d, pois = 3, 14 > 3.

Exemplo-16:

Joo colocou suas cabras num caminho para vender na feira. Tambm ps no caminho uma
quantidade de repolhos exatamente igual ao quadrado do nmero de cabras. Durante a viagem, cada
cabra comeu dois repolhos. Na feira, Joo vendeu 5 cabras e um nmero de repolhos. No final do
dia, ele observou com surpresa que o nmero de repolhos que tinha era igual ao quadrado o nmero
de cabras que tinha sobrado. Em seguida, colocou tudo o que no vendeu no caminho e retornou.
Mas durante a viagem, cada cabra comeu dois repolhos e chegando em sua casa, Joo tinha cabras
mas no mais repolho. Quantos repolhos Joo vendeu na feira?

Soluo
Depois de ler o problema (se preciso for, mais de uma vez, talvez at em voz alta), de forma a saber
exatamente quais as quantidades dadas, quais as condies dadas (hipteses) e o que se pede, vamos traar
um plano para a soluo.
A resposta 31.
Seja x a quantidade de cabras que Joo colocou no caminho. Assim, pela hiptese, conclumos que Joo
colocou x2 repolhos no caminho.
Ao chegar ao mercado, sobraram x2 2x repolhos. Se ele vendeu y repolhos sobraram x2 2x y. Mas,
como Joo vendeu 5 cabras, sobraram x 5 cabras.
20 Captulo 2. ESTRATGIAS

Assim, temos
x2 2x y = (x 5)2 ()
Durante a viagem de volta os x2 2x y repolhos foram comidos por x 5 cabras, que, por hipteses,
comiam na razo de 2 por cabra.
Logo, temos:
x2 2x y = 2(x 5) ()
De () e (), podemos concluir que

(x 5)2 = 2(x 5), com, (x 5) 6= 0.

Portanto, x 5 = 2 o que implica x = 7, que o nmero de cabras. Substituindo esse valor em (),
obtemos:
72 2 7 y = (7 5)2 = 49 14 y = 4 = y = 31
.
2.2 A busca do plano 21

2.2 A busca do plano


Uma das tcnicas de criatividade, chamada tempestade de ideias (em ingls brainstorming), geralmente
feita em grupo, revela que a quantidade gera a qualidade. O mtodo foi popularizado nos anos de 1930
pelo americano Alex Faicney Osborn. A fase em que voc procura um plano capaz de levar a soluo de um
problema uma situao parecida com a tempestade de ideia, s que exercida muitas vezes solitariamente
por voc. Esta fase do processo de resoluo do problema aquela em que deve nascer da sua cabea
muitas ideias, mesmo que possam parecer totalmente incuas.
Durante este processo, nenhuma ideia deve ser descartada ou julgada como errada ou absurda. As vezes, as
ideias em que voc menos aposta podem se revelar as mais apropriadas. Por isso, anote todas as suas ideias.
Se voc j tem algumas estratgias possveis para atacar o problema, a tarefa a seguir meditar sobre as
suas ideias e deixar que o seu subconsciente trabalhe, a seu gosto, esse amontoado de ideias que voc
preparou para ele. aconselhvel que voc faa uma lista das melhores ideias, a princpio sem mistur-las.
Explore cada ideia da lista com deciso e confiana, de forma ordenada, em paz, sem precipitaes.
Se, ao colocar em prtica uma ideia, lhe ocorrer outra, totalmente desligada da primeira, e que voc avalia
que pode lhe ajudar, no v desprez-la. Coloque-a na sua lista. Mas, tambm no desvie sua ateno da
que agora est explorando.
Um ponto importante que deve sempre ser lembrado: voc no pode desistir facilmente. Por outro lado, no
deve teimar demais s com uma ideia. Se as coisas complicarem demais, haver provavelmente outro
caminho. Um caminho que foi muito usado na histria de matemtica foi o da tentativa e erro, tentativa e
erro, tentativa e erro, voc no pode esquecer este fato.
Ao concluir sua resoluo, voc precisa ter certeza disso. Reveja sua soluo com cuidado.
Uma verdade: as meias ideias e as meias solues de pouco servem! preciso certificar de que, realmente,
voc chegou soluo.
Se voc conseguir resolver o problema, timo.
Se trabalhou horas a fio e no conseguiu vislumbrar uma soluo, no se preocupe. Muitas vezes se aprende
profundamente com os problemas que se tenta, com interesse, determinao e persistncia ...... e no se
consegue resolver, do que com os que se resolve primeira vista.
Um conselho deve ser lembrado: mais importante a qualidade do que a quantidade. No se apresse em
demasia. Ao concluir a soluo de um problema, preciso que voc reflita sobre o processo, para que tenha
uma ideia das dificuldades, dos becos sem sadas em que se meteu e, principalmente, como deve proceder no
futuro para resolver melhor outros problemas, parecidos ou no.
Uma reflexo sobe o nosso prprio processo de pensamento interessante na medida em que podemos tirar
bons proveitos para o futuro. Cada um tem seu prprio estilo de conhecimento. Visual ou analtico?
Depende muito da expresso verbal ou da forma escrita? Tem tendncia para o compromisso com ideia
nica, sem flexibilidade? Tem tendncia a pensar em crculo obsessivamente? Como se pode fomentar o
fluxo de ideias novas, variadas, originais? Reflexes como essas ajudam, a saber, que tipo de problemas
voc pode se ocupar com sucesso e em quais deles sua probabilidade de xito no to grande.

Exemplo-17:

Dois jogadores, A e B, disputam um jogo num tabuleiro 10 10, jogando alternadamente. O


jogador A possui uma lata de tinta azul e o jogador B uma lata de uma tinta vermelha. O jogador
A comea. Na sua vez de jogar, cada jogador escolhe uma linha ou coluna do tabuleiro que no
tenha sido previamente escolhida por qualquer um dos dois e pinta suas 10 casas com sua prpria
cor. Se qualquer uma destas casas escolhidas j estava pintada, a nova cor cobre a anterior. Aps
20 jogadas, quando no h mais as linhas e colunas disponveis, o jogo termina. Ento conta-se
o nmero de casas de cada cor e determina-se o vencedor de acordo com a seguinte regra: se o
nmero de casas vermelhas exceder a dez mais do que a quantidade de casas azuis, ento o jogador
B vence. Caso contrrio, o jogador A ganha.
Determinar se um dos dois jogadores tem uma estratgia vencedora e explicar a estratgia.
22 Captulo 2. ESTRATGIAS

Soluo

Depois de ler o problema (se preciso for, mais de uma vez, talvez at em voz alta), de forma a saber
exatamente quais as quantidades dadas, quais as condies dadas (hipteses) e o que se pede, vamos traar
um plano para a soluo.

Veremos, a seguir, que o jogador B possui uma estratgia vencedora. A estratgia do jogador B consiste
em, cada vez que o jogador A fizer seu movimento, ele escolhe uma linha ou coluna perpendicular
escolhida por A no movimento anterior. Para provar que esta estratgia vencedora, basta observar que
cada casa do tabuleiro pintada exatamente duas vezes, uma quando um jogador escolhe a linha em que
est localizada a casa e a outra quando um escolhe a coluna. Assim, aps as duas primeiras jogadas uma
nica casa fica com sua cor final e esta vermelha. Depois da terceira e quarta jogadas, trs novas caixas
atingem sua cor final, e das quais duas sero vermelhas e uma ser azul. Em geral, aps o movimentos
2k 1 e 2k existem 2k 1 novas casa que atingiram suas cores finais, das quais k 1 sero azuis e k
vo ser vermelhas. Como a cada duas jogadas a quantidade de casas vermelhas aumenta em um, ao final o
jogador B ter uma vantagem de 10 casas sobre o jogador A, vencendo o jogo.

Exemplo-18:

Dois jogadores, A e B, disputam o seguinte jogo, em que jogam alternadamente, usando um mao
de 2002 cartes. Cada carto tem nele escrito um nmeros inteiro de 1 a 2002 (um s nmero
em cada carto). Colocam-se os cartes sobre uma mesa, com a face virada para cima (os dois
jogadores vendo o nmero em cada carto). O jogador A comea. Uma jogada consiste em pegar
um carto da mesa, at que todos os cartes sejam retirados. Depois de retirados todos os cartes,
cada jogador soma os nmeros de seus cartes. Vence o jogo aquele jogador para o qual o ltimo
dgito da soma de todos os nmeros de seus cartes maior que o ltimo dgito da soma de todos os
nmeros dos cartes do seu adversrio. Imaginando que ambos os jogadores fazem suas melhores
jogadas, quem possui uma estratgia vencedora: A ou B?

Soluo

(Tournament of the Towns, Junior O level, fall 2002). Depois de ler o problema (se preciso for, mais de uma
vez, talvez at em voz alta), de forma a saber exatamente quais as quantidades dadas, quais as condies
dadas (hipteses) e o que se pede, vamos traar um plano para a soluo.

Nosso plano de soluo vai ser a estratgia do jogador A, que sempre vence. A estratgia do jogador A
consiste em formar todos os pares de cartes numerados com (k, 1000 + k), com k = 1, 2, 3, . . . , 1000,
mais o par (2001, 2002).
Agora, observe que todos os 1000 pares so formados com nmeros que possuem o mesmo ltimo dgito (o
mesmo dgito das unidades).
Na sua primeira jogada, o jogador A comea escolhendo para pegar o carto numerado com 2002. A
partir deste momento, sua estratgia buscar o outro elemento do par escolhido a cada vez que o jogador
B fizer sua escolha. Ento, eventualmente, o segundo jogador forado a pegar em algum momento o
carto numerado com 2001.
Em geral, se ainda tem carta no retiradas e todas as cartas escolhidas formam pares do tipo (k, 1000 + k),
na sua prxima jogada o jogador A retira qualquer carta numerada, digamos com k0 . Em seguida, o
segundo jogador ter que pegar, em algum momento, o carto com o qual formar o par com o carto
numerado com k0 . Isto decorre do fato de que, cada vez que o jogador B seleciona algum carto, o jogador
A escolhe o carto que forma o par com aquele escolhido por B.
No final do jogo, a soma dos cartes escolhidos pelo jogador A ser igual a

1000 1001
1 + 2 + 3 + . . . + 1000 + 2002 = + 2002 = 500 1001 + 2002,
2
que tem como ltimo dgito o 2, enquanto a soma dos nmeros dos cartes escolhido polo jogador B
termina em 1, o que garante a vitria de A.
2.2 A busca do plano 23

Exemplo-19:

Dois jogadores, A e B, disputam um jogo em que jogam alternadamente. De um baralho de cartas


extrai-se nove cartas numeradas de 2 a 10 e as coloca com a face para cima sobre a mesa. Uma
jogada consiste em retirar uma das cartas. O jogador A comea. Vence quem primeiro consegue
trs cartas cuja soma seja exatamente 18. Quem possui uma estratgia para no perder: A ou B?

Soluo

Depois de ler o problema (se preciso for, mais de uma vez, talvez at em voz alta), de forma a saber
exatamente quais as quantidades dadas, quais as condies dadas (hipteses) e o que se pede, vamos traar
um plano para a soluo.

O jogador A possui uma estratgia para no perder o jogo. Sua estratgia consiste em escrever os nove
nmeros como segue, formando um quadrado mgico, onde a soma dos nmeros em qualquer linha,
qualquer coluna e qualquer diagonal seja igual a 18:

3 10 5

8 6 4

7 2 9

Com esta estratgia, o jogador A joga como se estivesse disputando uma partida do jogo da velha. 1 Se
na sua primeira jogada ele escolhe a carta de nmero 6, a carta que est no centro do tabuleiro, ele no
perde o jogo, basta seguir atentamente a estratgia do jogo da velha.
Exemplo-20:

O nmero N o produto de k nmeros primos distintos (k 3). Dois jogadores, A e B, disputam


o jogo seguinte em que jogam alternadamente. O jogador A comea. Uma jogada consiste em
escrever no quadro-negro um divisor composto de N. Nenhum dos jogadores pode escrever N.
Tambm nunca pode aparecer dois nmeros relativamente primos ou dois nmeros, sendo um divisor
do outro. O primeiro jogador que no puder fazer seu movimento perde. Quem tem uma extratgia
vencedora: A ou B?

Soluo
(Olimpada de Matemtica de So Petersburg - 1997) Depois de ler o problema (se preciso for, mais de uma
vez, talvez at em voz alta), de forma a saber exatamente quais as quantidades dadas, quais as condies
dadas (hipteses) e o que se pede, vamos traar um plano para a soluo.
O jogador A possui uma estratgia vencedora. A estratgia do jogador A consiste em, no seu primeiro
movimento, escolher o nmero pq, com p, q nmeros primos distintos. Todo nmero subsequente ter de
1 Jogo da Velha O jogo da velha ou jogo do galo (no portugus europeu) um jogo e passatempo popular. um jogo de regras

extremamente simples, que no traz grandes dificuldades para seus jogadores e facilmente aprendido. Seu nome teria se originado
na Inglaterra, quando nos finais das tardes, mulheres se reuniram para conversar e bordar. As mulheres idosas, por no terem mais
condies de bordar em razo da fraqueza da viso, jogavam este jogo simples, que passou a ser conhecido como o da "velha". Porm,
sua origem teria sido ainda mais antiga. As regras do jogo so:
O tabuleiro uma matriz de trs linhas por trs colunas.
Dois jogadores escolhem uma marcao cada um, geralmente um crculo (O) e um xis (X).
Os jogadores jogam com as mos uma marcao por vez em uma lacuna que esteja vazia.
O objetivo conseguir 3 crculos ou trs xis em linha horizontal, vertical ou diagonal, e ao mesmo tempo, quando possvel,
impedir o adversrio de ganhar na prxima jogada.
Quando um jogador conquista o objetivo, costuma-se riscar os trs smbolos.
Quando um dos jogadores vence, o mesmo comea a outra partida
. (Referncia: htt ps : //pt.wikipedia.org/wiki/Jogod av elha)
24 Captulo 2. ESTRATGIAS

ser da forma pn ou pn, para algum nmero inteiro n > 1, relativamente primo com pq. Se o jogador B
escreve no quadro-negro pn, o jogador A escreve qn, e vice-versa. Com isso, o jogador A vence.
Exemplo-21:

Divide-se um tabuleiro de xadrez 8 8, veja figura abaixo, em K retngulos que no se sobrepem,


de acordo com as seguintes regras:
Cada retngulo formado unicamente juntando casas do tabuleiro (quadrados 1 1).
Cada retngulo possui a mesma quantidade de casas brancas e pretas.
No existem dois retngulos que sejam formados pela mesma quantidade de casas.

Qual o maior valor possvel para K?

Soluo

(Olimpada de Matemtica do Per) Depois de ler o problema (se preciso for, mais de uma vez, talvez at em
voz alta), de forma a saber exatamente quais as quantidades dadas, quais as condies dadas (hipteses) e
o que se pede, vamos traar um plano para a soluo.

Antes de estabelecer o plano para a soluo, observe que, pela segunda condio do problema, as reas
possveis dos K retngulos devem ser nmeros pares e, pela terceira condio, todos os nmeros pares
devem ser distintos.
Assim, se S a soma das reas dos K retngulos, S deve ser maior do que ou igual a soma dos primeiros
K nmeros pares positivos. Alm disso, S no mximo igual a 64, que quantidade de casas do
tabuleiro. Logo, temos:

2K(K + 1)
2 + 4 + 6 + . . . + 2K 64 64 K(K + 1) 64 = K 7.
2
Para concluir, basta mostrar que podemos realizar a diviso do tabuleiro com exatamente 7 retngulos
satisfazendo as condies do problema.
De fato, veja a figura abaixo, na qual cada nmero representa a quantidade de casas de cada retngulo:

2 14

4 12

6 10

16
2.2 A busca do plano 25

Exemplo-22:

Encontre todos os pares de nmeros inteiros positivos (x, y), para os quais

1 + 1996x + 1998y = xy.

Soluo

(Olimpada de Matemtica da Irlanda) Depois de ler o problema (se preciso for, mais de uma vez, talvez at
em voz alta), de forma a saber exatamente quais as quantidades dadas, quais as condies dadas (hipteses)
e o que se pede, vamos traar um plano para a soluo.

Vamos reescrever a expresso dada de uma forma equivalente para que possamos encontrar um plano que
nos leve soluo do problema.:

1+1996x+1998y = xy xy1996x1998y = 1 xy1996x1998y+19961998 = 1+19961998

(x 1998) (y 1996) = 1 + (1997 1) (1997 + 1) (x 1998) (y 1996) = 19972 .


Agora, observe que o nmero 1997 primo. Assim, temos, obrigatoriamente, que ter:

x 1998 = 1, 1997, 19972 e y 1996 = 1, 1997, 19972 ,

o que nos leva s seis solues possveis:

(1999, 19972 +1996), (1997, 19972 +1996), (3995, 3993), (1, 1), (19972 +1998, 1997), (19972 +1998, 1995).

Exemplo-23:

Prove que entre quaisquer dez pontos localizados na regio limitada por um crculo de dimetro
igual 5, existem dois desses pontos que esto distanciados um do outro por no mximo 2.

Soluo

(Olimpada de Matemtica do Japo) Depois de ler o problema (se preciso for, mais de uma vez, talvez at
em voz alta), de forma a saber exatamente quais as quantidades dadas, quais as condies dadas (hipteses)
e o que se pede, vamos traar um plano para a soluo.

Divida a regio limitada pelo crculo dado em nove partes: 1 crculo concntrico de raio 1 e mais 8
partes formadas pelas sub-regies determinadas pela parte de cada um dos setores congruentes que esteja
fora da regio limitada pelo crculo concntrico de raio 1, veja figura a seguir.

Agora, fcil ver que, dados 10 pontos localizados na regio limitada por um crculo de dimetro igual 5,
existem dois desses pontos dentro de uma mesma (destas nove) sub-regies e estes dois pontos esto a uma
distncia de no mximo 2, o que conclui a prova.
26 Captulo 2. ESTRATGIAS

Exemplo-24:

Escrevem-se em linha reta os nmeros inteiros de 1 a 37, de modo que cada nmero divide a soma
dos nmeros anteriormente escritos. Se o primeiro nmero escrito 37 e o segundo o 1, qual o
terceiro nmero escrito?

Soluo

(Olimpada de Matemtica da Rssia) Depois de ler o problema (se preciso for, mais de uma vez, talvez at
em voz alta), de forma a saber exatamente quais as quantidades dadas, quais as condies dadas (hipteses)
e o que se pede, vamos traar um plano para a soluo.

Observe que, pela hiptese do problema o terceiro nmero um divisor de 37 + 1 = 38 = 2 19. Logo, o
terceiro nmero escrito pode ser 2 ou 19. Agora, para dirimir a dvida, o segredo olhar para o ltimo
nmero escrito: x. Pela hiptese do problema, temos que x divide a expresso:

37 (37 + 1)
1 + 2 + 3 + . . . + 37 = = 37 19.
2
Como 37 e 19 so nmeros primos e 1 < x < 37, segue que x = 19. Logo, o terceiro nmero no pode
ser 19. Portanto, o terceiro nmero tem de ser 2.
Exemplo-25:

Divide-se em tringulos a regio limitada por um polgono regular de 1997 lados, usando diagonais
que no se interceptam no interior da regio. Prove que no mnimo um destes tringulos acutngulo.

Soluo

(Olimpada de Matemtica da Rssia) Depois de ler o problema (se preciso for, mais de uma vez, talvez at
em voz alta), de forma a saber exatamente quais as quantidades dadas, quais as condies dadas (hipteses)
e o que se pede, vamos traar um plano para a soluo.

Basta observar que o crculo circunscrito ao polgono dado tambm o crculo circunscrito a cada um dos
tringulos. Como o polgono possui um nmero mpar de lados, segue que o centro do crculo circunscrito
no pertence a qualquer uma das diagonais. Isto , o centro do crculo est localizado no interior de um dos
tringulos. Isto suficiente para que possamos concluir que o tringulo acutngulo.
Exemplo-26:

Escrevem-se num quadro negro os nmeros inteiros de 1 at 1000. Dois jogadores, A e B,


disputam o jogo seguinte, em que fazem seus movimentos alternadamente. O jogador A comea.
Uma jogada consiste em apagar um dos nmeros escritos no quadro. O jogo termina quando
restarem dois nmeros. O jogador A vence se a soma dos dois nmeros restantes for divisvel por
3, caso contrrio, o jogador B ganha. Qual dos dois jogadores possui uma estratgia vencedora?

Soluo

(Olimpada de Matemtica da Rssia) Depois de ler o problema (se preciso for, mais de uma vez, talvez at
em voz alta), de forma a saber exatamente quais as quantidades dadas, quais as condies dadas (hipteses)
e o que se pede, vamos traar um plano para a soluo.

O jogador B possui uma estratgia vencedora: se o jogador A apaga o nmero x, ento o jogador B
apaga o nmero 1001 x. Portanto, os ltimos dois nmeros somaro 1001, que no divisvel por 3.
2.2 A busca do plano 27

Exemplo-27:

Escreve-se um nmero em cada uma das 16 casas de um tabuleiro 4 4 . Para qualquer casa, a
soma dos nmeros escritos nas casas que tem um lado comum com ela igual a 1. Determine a
soma dos 16 nmeros escritos nas casas do tabuleiro.

Soluo
(Torneio das Cidades - 2000) A resposta 6. Depois de ler o problema (se preciso for, mais de uma vez,
talvez at em voz alta), de forma a saber exatamente quais as quantidades dadas, quais as condies dadas
(hipteses) e o que se pede, vamos traar um plano para a soluo.
A estratgia pintar as casas do tabuleiro alternadamente branca e marrom. Cada casa branca vizinha,
com um lado comum, de uma das casas marrons assinaladas, veja figura a seguir.

Observe que, pela hiptese do problema, podemos concluir que a soma dos nmeros nas casas brancas
igual a 3. Resta mostrar que a soma dos nmeros nas casa marrons igual a 3. Para isso, preencha o
centro do tabuleiro com 0 e as demais casas como na figura a seguir.

w z y x

x 0 0 w

y 0 0 z

z w x y

Como x + z = 1 e y + w = 1, segue que a soma dos nmeros escritos nas casas do tabuleiro igual a
3(x + z) + 3(y + w) = 3 1 + 3 1 = 3 + 3 = 6.

Exemplo-28:

Sejam n um nmero inteiro positivo. Divide-se um tabuleiro retangular 4 n em retn-


gulos 2 1 ou 1 2 (como se o tabuleiro fosse coberto com peas de um domin, sem
superposies e sem deixar buracos, com cada domin cobrindo precisamente duas casas do
tabuleiro). Em seguida, pinta-se de vermelho todos os pontos do tabuleiro que so vrtices de algum
dos retngulos 2 1 ou 1 2. Qual a menor quantidade de pontos vermelhos que se pode obter?

Soluo
(Olimpada de Matemtica do Per) Depois de ler o problema (se preciso for, mais de uma vez, talvez at em
voz alta), de forma a saber exatamente quais as quantidades dadas, quais as condies dadas (hipteses) e
o que se pede, vamos traar um plano para a soluo.
28 Captulo 2. ESTRATGIAS

Observe que, como n um inteiro positivo qualquer, nossa resposta vai ser dada em funo de n. Assim,
vamos chamar de f (n) o nmero mnimo de pontos vermelhos que podemos obter ao cobrir um tabuleiro
4 n com peas de um domin, satisfazendo s hipteses do problema. Fazendo um desenho, fcil ver
que:
f (1) = 6, f (2) = 9, f (3) = 12, f (4) = 15 e f (5) = 18.

n=1 f(1) = 6

n=2 f(2) = 9


n=3 f(3) = 12

n=4 f(4) = 15

n=5 f(5) = 18

Nosso objetivo mostra por induo que:

f (2k) = 5k + 2 e f (2k + 1) = 5k + 8, para todo nmero inteiro k 2. ()

Para isso, seja n 2, e suponha que j conhecemos os valores de f (1), f (2), f (3), . . . , f (2n), f (2n + 1).
Agora, considere um tabuleiro 4 (2n + 2) coberto completamente por domins, nas hipteses do
problema. Podem ocorrer trs situaes:
2.2 A busca do plano 29

A ltima linha do tabuleiro, contada de baixo para cima, possui como cobertura dois domins
colocados horizontalmente.
Nas duas ltimas linhas, contada de baixo par a cima, existem 4 peas de domins colocadas
verticalmente.
Nas duas ltimas linhas do tabuleiro, contadas de baixo para cima, hajam 1 pea de domin
colocada horizontalmente 2 colocadas verticalmente.

Vamos provar nosso objetivo () para cada um dos casos acima.

A ltima linha do tabuleiro, contada de baixo para cima, possui como cobertura dois domins
colocados horizontalmente, veja figura a seguir.

.... .... .... ....


.. .. .. ..
2n + 1

Portanto, esta ltima linha, contada de baixo para cima, gera 3 novos pontos vermelhos, o que
significa que existem pelo menos

3 + f (2n + 1) = 3 + 5n + 8 = 5n + 11 pontos vermelhos.

Nas duas ltimas linhas, contada de baixo par a cima, existem 4 peas de domins colocadas
verticalmente, veja figura ilustrativa a seguir .
30 Captulo 2. ESTRATGIAS

.... .... .... ....


.. .. .. ..
2n

Portanto, estas duas ltimas linhas, contada de baixo para cima, geram 5 novos pontos vermelhos, o
que significa que existem pelo menos
5 + f (2n) = 5 + 5n + 5 = 5n + 10 pontos vermelhos.
Nas duas ltimas linhas do tabuleiro, contadas de baixo para cima, hajam 1 pea de domin
colocada horizontalmente 2 colocadas verticalmente, veja figuras ilustrativas a seguir.


2n + 2 - k

.... .... .... ....


.. .. .. ..

Assim, teremos um total de pontos vermelhos no mnimo igual a:


k k+1
4 + 3(k 1) + f (2k + 2 k) = 5n + 11 + , se k par ou 5n + 11 + , se k mpar.
2 2
2.2 A busca do plano 31

Observe que ambos os valores so maiores do que 5n + 10.


Com isso, conclumos que f (2n + 2) 5n + 10, que o menor obtido ao analisarmos os dois casos.
Colocando todas peas do domin na horizontal fcil ver que existem um total de 5n + 10 pontos
vermelhos. Portanto, f (2n + 2) = 5n + 10.
De forma anloga, fcil ver que f (2n + 3) = 5n + 13, o que completa a prova.

Exemplo-29:

Diga, justificando, se possvel escolher 1000 pontos do plano de tal modo que no mnimo 6000
distncias entre dois deles sejam iguais.

Soluo

(BW - 2003) Depois de ler o problema (se preciso for, mais de uma vez, talvez at em voz alta), de forma a
saber exatamente quais as quantidades dadas, quais as condies dadas (hipteses) e o que se pede, vamos
traar um plano para a soluo.

A resposta sim. Comece com uma configurao de 4 pontos e 5 distncias, veja figura a seguir.


d d

Agora tome a figura acima e mais duas cpias congruentes a ela, obtidas por movimentos paralelos (e no
coincidentes), veja figura a seguir.


d d



d
d


d d

Neste caso, tem-se 3 4 = 12 pontos e 3 5 + 12 = 27 distncias. Agora junte mais trs cpias da figura
acima, obtendo:
3 12 = 36 pontos e 3 27 + 36 = 117 dist ancias.

Agora, forme uma figura com 9 figuras congruentes a figura inicial, obtidas dela por movimentos paralelos
(e no coincidentes). Desse modo, temos

3 36 = 108 pontos e 3 117 + 108 = 459 dist ancias.



32 Captulo 2. ESTRATGIAS

Procedendo desta maneira, sempre juntando mais 3 cpias da figura inicial, vamos obtendo gradativamente:

3 108 = 324 pontos e 3 459 + 324 = 1701 dist ancias.


3 324 = 972 pontos e 3 1701 + 972 = 6075 dist ancias.



2.3 Procure semelhanas com outros problemas 33

2.3 Procure semelhanas com outros problemas


Como no h nada de novo debaixo do cu, conveniente procurar semelhanas do problema dado com
outros que voc j conhece.
Pergunte a si mesmo: o que que esse problema me faz lembrar?
Se seu problema for genrico, tente primeiro alguns casos particulares. Caso o problema envolva a
geometria tridimensional, voc poder tentar primeiro um problema bidimensional anlogo.

Exemplo-30:

Resolva a equao x4 5x2 + 6 = 0.

Soluo

Depois de ler o problema (se preciso for, mais de uma vez, talvez at em voz alta), de forma a saber
exatamente quais as quantidades dadas, quais as condies dadas (hipteses) e o que se pede, vamos traar
um plano para a soluo.

Se a equao dada parece difcil, troque x2 por t, obtendo a equao

t 2 5t + 6 = 0 (),

() so t = 2
que mais fcil de resolver. Assim, as solues da equao
2
2 e t = 3, o que implica x = 2
ou x = 3. Portanto, as solues da equao dada so: 2 e 3.
Exemplo-31:

No vrtice A de uma caixa retangular fechada temos uma formiga e no vrtice B temos outra
formiga, veja Figura a seguir.

A formiga na posio A vai se deslocar sobre a caixa para encontrar a formiga na posio B.
Qual o caminho mais curto sobre a caixa, ligando o ponto A ao ponto B, que a formiga deve
escolher para fazer seu deslocamento?

Soluo

Depois de ler o problema (se preciso for, mais de uma vez, talvez at em voz alta), de forma a saber
exatamente quais as quantidades dadas, quais as condies dadas (hipteses) e o que se pede, vamos traar
um plano para a soluo.

O que que esse problema me faz lembrar?

Se a questo fosse num retngulo, figura plana, em vez de uma caixa retangular, figura tridimensional, a
questo seria resolvida simplesmente procurando o menor caminho sobre o plano ligando dois pontos, que
sabemos ser um segmento de reta. O que temos que fazer tornar a caixa tridimensional um objeto
bidimensional. Como fazer isso? Abrindo a caixa, marcando os dois pontos e traando o segmento de reta
ligando os pontos A e B, veja figura a seguir.
34 Captulo 2. ESTRATGIAS

Depois disso, recompomos a caixa, mostrando o menor caminho ligando os pontos A e B, veja figura a
seguir.

Exemplo-32:

Qual a soma dos n termos da sries 7 + 77 + 777 + 7777 + ?

Soluo

Depois de ler o problema (se preciso for, mais de uma vez, talvez at em voz alta), de forma a saber
exatamente quais as quantidades dadas, quais as condies dadas (hipteses) e o que se pede, vamos traar
um plano para a soluo.

Se em vez de 7 + 77 + 777 + 7777 + tivssemos

9 + 99 + 999 + 9999 + ?

Neste caso, reescreveramos a sries como:

9 + 99 + 999 + 9999 + = (10 1) + (100 1) + (1000 1) + (10000 1) + =

= (10 + 100 + 1000 + 10000 + ) n


10n+1 10
As potncias de 10 formam uma progresso geomtrica de n termos cuja soma igual a 9 . Assim,

9 + 99 + 999 + 9999 + =
10n+1 10
= (10 1) + (100 1) + (1000 1) + (10000 1) + = n.
9
7
Agora, observe que para encontrar a soma pedida, basta observar que 7 igual a 9 de 9, o que implica que
a soma pedida igual a
7
7 + 77 + 777 + 7777 + = 7(1 + 11 + 111 + 1111 + ) = 9(1 + 11 + 111 + 1111 + ) =
9
2.3 Procure semelhanas com outros problemas 35
 n+1 
7 10 10 7
= (9 + 99 + 999 + 9999 + ) = n = (10n+1 9n 10).
9 9 81

Exemplo-33:

Numa sala de aula existem 25 alunos sentados em 5 filas com 5 lugares em cada fila. Um dia o
professor pede para cada aluno mudar seu lugar como segue: cada um deve mover para um assento
para a frente ou para trs ou um assento esquerda ou direita - movimento em diagonal no
permitido.
possvel que todos os 25 alunos sigam estas instrues?

Soluo

Depois de ler o problema (se preciso for, mais de uma vez, talvez at em voz alta), de forma a saber
exatamente quais as quantidades dadas, quais as condies dadas (hipteses) e o que se pede, vamos traar
um plano para a soluo.

A abordagem tradicional tentar vrios movimentos. Isso geralmente no resolve o problema e causa certa
frustrao. A ideia aqui resolver um problema anlogo mais simples. Assim, faa um desenho da sala e
numere os assentos, veja figura a seguir.

21 22 23 24 25

16 17 18 19 20

11 12 13 14 15

6 7 8 9 10

1 2 3 4 5

Agora, observe que, dentre os nmeros inteiros de 1 a 25, temos 13 nmeros mpares e somente 12
nmeros pares. Com os movimentos permitidos, cada aluno passa de uma cadeira de nmero mpar para
uma cadeira de nmero par, e vice-versa. Como a quantidade de nmeros pares difere da quantidade de
nmeros mpares, vai haver um aluno que que no pode se movimentar.
Portanto, impossvel que todos os 25 alunos sigam as instrues do professor.
Exemplo-34:

Encontre o valor da expresso:

S = 1002 992 + 982 972 + 962 952 + . . . + 42 32 + 22 12 . ()

Soluo

Depois de ler o problema (se preciso for, mais de uma vez, talvez at em voz alta), de forma a saber
exatamente quais as quantidades dadas, quais as condies dadas (hipteses) e o que se pede, vamos traar
um plano para a soluo.

No confortvel efetuar as operaes indicao, pela quantidade de nmeros existente. Em vez disso,
usaremos a expresso para a diferena de dois quadrados:

x2 y2 = (x + y) (x y)
36 Captulo 2. ESTRATGIAS

Assim, a soma () pode ser escrita como:

S = (100 + 99) (100 99) + (98 + 97) (98 97) + . . . + (4 + 3) (4 3) + (2 + 1) (2 1). ()

Agora, observe que todas as expresses do segundo parnteses de cada parcela de S em () igual a 1.
Portanto, temos S = 100 + 99 + 98 + 97 + . . . + 4 + 3 + 2 + 1 = 100(100+1
2 = 50 101 = 5050.
Exemplo-35:

Para que valores de n o nmero 28 + 211 + 2n um quadrado perfeito?

Soluo

(Olimpada de Matemtica do Cone Sul) Depois de ler o problema (se preciso for, mais de uma vez, talvez
at em voz alta), de forma a saber exatamente quais as quantidades dadas, quais as condies dadas
(hipteses) e o que se pede, vamos traar um plano para a soluo.

Podemos observar que:


28 + 211 + 2n = (24 )2 + 2 24 26 + 2n
Agora, observe que o nmero do lado direito da igualdade acima muito parecida com a expresso do
quadrado de um binmio:
(a + b)2 = a2 + 2ab + b2 .
Assim, tomando a = 24 , b = 26 , teremos: (24 + 26 )2 = (24 )2 + 2 24 26 + 212 , o que nos leva a concluir
que n = 12.
Resta-nos a pergunta: n = 12 a nica soluo?
A resposta sim. Suponhamos que exista outra soluo. Assim, suponha que existe um nmero inteiro k
para o qual
28 + 211 + 2n = k2 .
Neste caso, podemos escrever

28 + 211 + 2n = 28 + 23 28 + 2n = k2 9 28 + 2n = k2 2n = k2 32 28 2n = (k + 3 24 )(k 3 24 ).

Assim, claro que os fatores (k + 3 24 ) e (k + 3 24 ) so obrigatoriamente potncia de 2. Portanto,


existem nmeros naturais a e b, com a + b = n, tais que

2a = k + 3 24 e 2b = k 3 24 .

Logo, 2a 2b = 2 3 24 2b (2ab 1) = 3 25 .
Portanto, 2b = 25 e 2ab 1 = 3, o que resulta em b = 5, 2ab = 4 a = 7. Portanto, n = a + b = 12,
o que conclui a prova.
2.4 Comear pelo fcil torna fcil o difcil 37

2.4 Comear pelo fcil torna fcil o difcil


Talvez o problema seja complicado porque h muitos elementos. Tente tornar o problema mais fcil.
Construa um, menos complicado, com menos dados. Talvez essa situao venha lhe revelar algo que facilite
a soluo do problema mais complexo.
Exemplo-36:

Prove que, se a, b, c so nmeros reais, ento

a2 b2 + b2 c2 + a2 c2 abc(a + b + c)

Soluo

Em vez de resolver a questo diretamente, mais confortvel pensar no problema seguinte, que mas
simples e equivalente ao problema que queremos resolver:

Se x, y, z so nmeros reais, prove que:

x2 + y2 + z2 xy + xz + yz00 .

fcil ver que, a soluo do nosso problema mais simples decorre do fato de que a soma de trs quadrados
perfeitos sempre maior do que ou igual a zero:

(x y)2 + (y z)2 + (x z)2 0 = 2(x2 + y2 + z2 ) 2(xy + xz + yz) 0 = x2 + y2 + z2 xy + xz + yz.

Agora, usamos nosso problema mais simples para resolver o problema dado. Para isso, basta escrever

x = ab, y = ac, z = bc.

Assim, segue imediatamente a prova da desigualdade dada.


Exemplo-37:

possvel encontrar 2016 nmeros inteiros positivos distintos, cada um deles sendo um quadrado
perfeito, tal que a soma de todos eles seja tambm um quadrado perfeito?

Soluo

Em vez de resolver a questo diretamente, mais confortvel pensar no problema seguinte, que mas
simples e permite a soluo do problema que queremos resolver:

Como veremos a resposta sim.


Se o problema fosse encontrar dois nmeros inteiros positivos quadrados perfeitos cuja soma fosse um
quadrado perfeito, usaramos o teorema de Pitgoras:

32 + 42 = 52 .

Agora, vamos ver que esta situao mais simples nos revela uma sada para o problema mais complexo. De
fato, multiplique cada lado da ltima igualdade por 52 , obtendo

32 .52 + 42 .52 = 52 .52

e substitua 52 na primeira parcela do lado esquerdo por 32 + 42 . Assim, obtemos

32 (32 + 42 ) + 42 52 = 52 32 .32 + 32 .42 + 42 .52 = 52 .52 .

Novamente, multiplicando cada lado da ltima igualdade por 52 , obtemos

32 .32 .52 + 32 .42 .52 + 42 .52 .52 = 52 .52 .52


38 Captulo 2. ESTRATGIAS

e usando na primeira parcela esquerda a substituio de 52 por 32 + 42 , obtemos:


32 .32 .32 + 32 .32 .42 + 32 .42 .52 + 42 .52 .52 = 52 .52 .52 .
Continuando esse processo, encontraremos 2016 nmeros inteiros positivos distintos, todos quadrados
perfeitos, tal que a soma deles seja tambm um quadrado perfeito.
Exemplo-38:

Calcular a rea lateral do tronco de cone de altura h e raios R e r, veja figura a seguir.

Soluo

O tronco de cone uma superfcie tridimensional. Em vez de resolver a questo diretamente, mais
confortvel pensar num problema no plano. Assim, cortamos o tronco de cone ao longo de um segmento de
reta, e colocamos sobre o plano, obtendo um trapzio, veja figura a seguir:

r
2r 2r

R
s
2R 2R

Agora, usamos que a rea, S, do trapzio igual a


(comprimento da base maior) + (comprimento da base menor)
S= (altura).
2
O comprimento da base maior igual ao comprimento do crculo maior: 2R.
O comprimento da base menor igual ao comprimento do crculo menor: 2r.
Resta calcular a altura do trapzio, que igual ao comprimento do segmento da geratriz do tronco do cone.
Seja g o comprimento do lado geratriz do tronco do cone, veja figura a seguir

h h g

R R-r
2.4 Comear pelo fcil torna fcil o difcil 39

Usando a figura acima e o Teorema de Pitgoras, temos


q
g2 = h2 + (R r)2 = g = h2 + (R r)2 .

Portanto, a rea lateral, S, do tronco de cone de altura h e raios R igual a


2R + 2r
q q
S= h2 + (R r)2 = (R + r) h2 + (R r)2 .
2

Exemplo-39:

Se x, y, p so nmeros reais positivos, com 0 < p < 1, prove a desigualdade

(x + y) p < x p + y p

Soluo
Observe que, se trocarmos x e y por tx e ty, com t positivo, respectivamente, a desigualdade continua
valendo. Ou seja, a desigualdade invariante pela transformao que leva x em tx, y em ty. Isto
significa que, sem perda de generalidade, podemos supor que x + y = 1. Mas, se x + y = 1, com x, y
positivos, a desigualdade bvia, pois, neste caso, 0 < x < 1 e 0 < y < 1, implicando que x < x p e
y < y p . Portanto a desigualdade verdadeira.
Exemplo-40:

Resolva a seguinte equao:

(x2 4x + 2014) (x2 4x + 2016) + 20152 = 0

Soluo

Em vez de resolver a questo diretamente, mais confortvel pensar numa alternativa que possa simplificar
a expresso do lado esquerdo da equao dada.

Para isso, vamos introduzir uma nova varivel, com a qual possamos simplificar os dados do problema.
Assim, seja
y = x2 4x 1.
Substituindo y na equao dada, obtemos:

(x2 4x + 2014) (x2 4x + 2016) + 20152 = 0 (y + 2015) (y 2015) + 20152 = 0

y2 20152 + 20152 = 0 y2 = 0 y = 0.
Portanto,
y = 0 x2 4x 1 = 0 x = 2 5.
40 Captulo 2. ESTRATGIAS

2.5 Experimente, procure algo que seja invariante


Muita matemtica foi feita por tentativa, errando, corrigindo, aperfeioando, avanando.
As vezes pode ser vantajoso introduzir no problema algo novo, um auxlio extra, para que facilite voc na
percepo entre o que foi dado e o que foi pedido.
Exemplo-41:

Um jogo consiste de 9 botes luminosos (de cor verde ou vermelho) dispostos num quadrado da
maneira seguinte:

1 2 3

4 5 6

7 8 9

Apertando um boto do bordo do quadrado, mudam de cor esse boto e todos seus vizinhos.
Apertando o boto do centro, mudam de cor seus 8 vizinhos mas ele no.
Os exemplos seguintes mostram com crculos pretos as luzes que mudam de cor ao serem pressionado
o boto que se indica.




Boto 1 Boto 2 Boto 5

possvel (apertando sucessivamente alguns botes) acender todas as luzes com cor verde, se
inicialmente estavam todas acesas com a cor vermelha? Justifique a resposta.

Soluo
(Olimpada de Matemtica do Cone Sul) Inicialmente, observamos que, ao apertar um boto qualquer,
mudam de cor um nmero par de botes, de acordo com a tabela seguinte.

Boto N o Mudam de Cor


1, 3, 7 ou 9 4 botes
2, 4, 6 ou 8 6 botes
5 8 botes

Assim, a quantidade de botes que mudam da cor vermelho para a cor verde possui a mesma paridade que a
quantidade de botes que mudam de verde para vermelho. Desta maneira, sempre haver uma quantidade
par de botes verdes, de modo que nunca se poder chegar a situao de se ter 9 botes verdes (em todo o
quadrado), pois 9 um nmero mpar.
De fato, seja an a quantidade de botes com a cor verde depois de n apertos sucessivos. Se v a
quantidade de botes que mudam de vermelho para verde e r a quantidade de botes que mudam de
verde para vermelho, no passo seguinte teremos

an+1 = an + v r.

Mas, como (v r) um nmero par, segue que an+1 e an tem a mesma paridade. Como inicialmente
ao = 0, implica que an par para todo valor de n. Portanto, an 6= 0.
2.5 Experimente, procure algo que seja invariante 41

Exemplo-42:

Tem-se quarenta e trs pedaos de palitos, cujos comprimentos so 1, 2, 3, , 42, 43 centmetros,


respectivamente.
Diga, justificando, se possvel formar um quadrado usando todos estes pedaos, sem quebrar
qualquer um deles, nem sobrepor dois ou mais deles.
E se em vez de um quadrado for um retngulo?

Soluo
Aqui conveniente observar que

43 (43 + 1)
1 + 2 + 3 + + 42 + 43 = = 43 22 = 946.
2

Se fosse possvel formar um quadrado, de lado medindo x, usando todos estes pedaos, sem quebrar
qualquer um deles, nem sobrepor dois ou mais deles, teramos necessariamente que o permetro do
quadrado seria igual a 946. Ou seja, 4x = 946, o que impossvel, pois x tem de ser um nmero inteiro e
o nmero 946 no divisvel por 4.
Se em vez de um quadrado for um retngulo, com lados medindo a e b centmetros, teramos o permetro
do retngulo como sendo igual a 2a + 2b = 946, ou seja a + b = 473. Neste caso, a resposta sim. Se os
quatro lados do retngulo medem a, a, b, b, poderamos tomar:

a = 1 + 2; e a = 3, e

b = (4 + 43) + (6 + 41) + (8 + 39) + (10 + 37) + + (19 + 29) + (20 + 27) + (22 + 25)

b = (5 + 42) + (7 + 40) + (9 + 38) + (11 + 36) + + (19 + 28) + (21 + 26) + (23 + 24).

Exemplo-43:

Um tabuleiro 10 10 pode ser coberto por 25 domins de dimenses 1 4?

Soluo
Aqui o invariante mais bvio a rea, que infelizmente no ajuda na soluo do problema. Por outro lado,
podemos escrever um nmero em cada um dos 100 quadrados unitrios do tabuleiro com a propriedade
que, no importa como ns colocamos um dos domins cobrindo 4 quadrados, a soma dos nmeros em
quatro quadrados coberto seja igual a zero; no entanto, a soma de todos os 100 nmeros em todo o
tabuleiro no zero. Ento, obviamente, a cobertura impossvel.
Como associar os nmeros aos quadrados unitrios do tabuleiro do modo com prevemos acima?
Defina os valores an , para n = 1, 2, 3, , 9, 10, da seguinte maneira:

a1 = 1 a2 = 1 a3 = 1 a4 = 3 a5 = 1 a6 = 1 a7 = 1 a8 = 3 a9 = 1 a10 = 1

Nesta sequncia, qualquer bloco consecutivo de quatro termos tem soma igual a 0, mas a soma de todos os
dez no 0. Agora, escrevemos no quadrado unitrio do tabuleiro que est na linha m e coluna n o nmero
am an , que nos sugere uma pintura para o tabuleiro usando trs cores distintas, veja figura a seguir.
42 Captulo 2. ESTRATGIAS

1 1 1 -3 1 1 1 -3 1 1
1 1 1 -3 1 1 1 -3 1 1
1 1 1 -3 1 1 1 -3 1 1
-3 -3 -3 9 -3 -3 -3 9 -3 -3
1 1 1 -3 1 1 1 -3 1 1
1 1 1 -3 1 1 1 -3 1 1
1 1 1 -3 1 1 1 -3 1 1
-3 -3 -3 9 -3 -3 -3 9 -3 -3
1 1 1 -3 1 1 1 -3 1 1
1 1 1 -3 1 1 1 -3 1 1

Exemplo-44:

Trs mquinas caa-niqueis imprimem pares de inteiros positivos em cartes. As mquinas caa-
nqueis trabalham da seguinte maneira. A primeira mquina caa-nquel depois de ler um carto
(a, b) imprime um novo carto (a + 1, b + 1); a segunda mquina caa nquel depois de ler um
carto (a, b) imprime um novo carto ( a2 , b2 ). Esta mquina s funciona se os nmeros a e b
forem pares. A terceira mquina caa-nquel, depois de ler dois cartes (a, b) e (b, c) imprime um
novo carto (a, c). Alm disso, as mquina caa-nquel devolvem todos os cartes lidos.
possvel comear com um carto (4, 18) e chegar num carto (1, 100)?

Soluo
(AUMO - 1978) A resposta no. Para qualquer carto (a, b) chame D(a, b) a diferena a b.
Seja S a coleo de todos os cartes obtidos a partir do carto (4, 18) por meio de operaes
consecutivas das mquinas caa-nquel.
Vamos mostrar que D(a, b) divisvel por 7, para cada (a, b) S. Isto , o resto da diviso de D(a, b)
por 7 um invariante.
Para mostrar isso, vamos usar induo sobre o nmero de sucessivos usos das mquinas.
Observe que D(4, 18) = 4 18 = 14. Logo, o resto da diviso de D(4, 18) por 7 zero:
14 = (2) 7 + 0. Vamos supor que depois do nsimo uso das mquinas tenhamos um carto (a, b),
com o resto da diviso de D(a, b) por 7 igual a zero.
Vamos considerar o (n + 1)simo uso das mquinas caa-niquel. Existem trs casos possveis:
Usando a primeira mquina, depois de colocar o carto (a, b), obtemos o carto (a + 1, b + 1).
Neste caso, D(a + 1, b + 1) = (a + 1) (b + 1) = a b, que, por hiptese de induo, divisvel por
7.
Usando a segunda mquina, depois de colocar o carto (a, b), obtemos o carto ( a2 , 2b ).
Neste caso, D( a2 , b2 ) = a2 b2 ) = 12 (a b), que, por hiptese de induo, divisvel por 7.
Usando a terceira mquina, depois de colocar os cartes (a, b) e (c, d), obtemos o carto (a, c).
Neste caso, D(a, c) = a c = a b + b c = (a b) + (b c), que, por hiptese de induo,
divisvel por 7, por ser soma de dois nmeros mltiplos de 7.
Assim, em todos os trs casos, o resto da diviso de D(a, b) por 7 no novo carto nulo.
O mesmo verdade para o caso do carto obtido a partir de (4, 18) nas trs mquinas caa-niquel. Mas,
D(1, 100) = 1 100 = 99 = (15) 7 + 6. Portanto, D(1, 100) deixa resto 6 na diviso por 7,
consequentemente no possivel obter o carto (1, 100) a partir do carto (4, 18).
2.6 Faa um desenho e, dependendo da situao, pinte s cores. 43

2.6 Faa um desenho e, dependendo da situao, pinte s cores.


Muitos de ns pensamos melhor com um desenho, esquema, imagem, do que com palavras. Diz o ditado
popular: uma imagem vale mais do que mil palavras.
Logo, sempre que puder, faa um esquema auxiliar, um desenho, pinte s cores. Essas atitudes, quase
sempre revelam caminhos surpreendentemente elegantes e fceis de comprovar.
Exemplo-45:

Considere um tabuleiro de xadrez 8 8 e 32 domins de dimenso 2 1. Os domins podem ser


arranjados sobre o tabuleiro de modo a cobri-lo inteiramente, cada domin cobrindo perfeitamente
duas casas e no havendo sobreposio de domins. Do tabuleiro, retiram-se duas casas, situadas
nos cantos do tabuleiro de uma mesma diagonal, veja Figura a seguir.

Diga, justificando, se 31 domins cobrem completamente o tabuleiro reduzido.

Soluo

A resposta no.

A ideia aqui pintar o tabuleiros de modo que as casas fiquem alternadamente pretas e brancas, veja figura
a seguir.

Observe que os dois quadrados situados nos cantos do tabuleiro possuem a mesma cor e que cada domin
por inteiro cobre sempre uma casa de cada cor, independente de como voc o coloca sobre o tabuleiro na
horizontal ou vertical. Assim, se a cobertura fosse possvel deveramos ter no tabuleiro a mesma quantidade
de casas de cada cor, o que no ocorre com a retirada de duas casas de mesma cor.
Portanto, 31 domins no cobriro completamente o tabuleiro reduzido.
44 Captulo 2. ESTRATGIAS

Exemplo-46:

Escreve-se um nmero em cada uma das 16 casas de um tabuleiro 4 4 . Para qualquer casa,
a soma dos nmeros escritos nas casas que tem um lado comum com ela igual a 1. Determine a
soma dos 16 nmeros escritos nas casas do tabuleiro.

Soluo (Torneio das Cidades - 2000) A resposta 6.


Pinte as casas do tabuleiro alternadamente branca e marron. Cada casa branca vizinha, com um lado
comum, de uma das casas marrons assinaladas, veja figura a seguir.

Observe que, pela hiptese do problema, podemos concluir que a soma dos nmeros nas casas brancas
igual a 3. Resta mostrar que a soma dos nmeros nas casa marrons igual a 3. Para isso, preencha o
centro do tabuleiro com 0 e as demais casas como na figura a seguir.

w z y x

x 0 0 w

y 0 0 z

z w x y

Como x + z = 1 e y + w = 1, segue que a soma dos nmeros escritos nas casas do tabuleiro igual a
3(x + z) + 3(y + w) = 3 1 + 3 1 = 3 + 3 = 6.

Exemplo-47:

Coloca-se uma ficha em cada um dos seis crculos da figura a seguir.

O jogador pode escolher quaisquer duas fichas e moviment-las em direes opostas pelo mesmo
nmero de passos. O objetivo colocar todas as fichas num mesmo crculo, formando uma pilha.
(a) O jogador consegue alcanar seu objetivo? Se ele consegue, descreva um mtodo. Se no, prove
que isto impossvel.
(b) E se, em vez de seis, fossem 10 crculos?
2.6 Faa um desenho e, dependendo da situao, pinte s cores. 45

Soluo

(a) impossvel.
Pintemos os crculos alternadamente de branco e preto, veja figura a seguir.

Seja S a quantidade de fichas nos crculos brancos em determinado instante. Inicialmente, S = 3. Observe
que, aps cada movimento do jogador, ou S aumenta de 2, ou S diminui de 2 ou S no se altera. Ora,
isto significa que S permanece um nmero mpar durante todo o jogo. Por outro lado, se todas as fichas
ficarem num mesmo crculo, formando uma pilha com seis fichas, teremos: ou S = 6 ou S = 0, que so
dois nmeros pares. Portanto, impossvel colocar todas as fichas num mesmo crculo.
(b) Neste caso, possvel.
Observe que n = 10 = 2 5. Ou seja, n = 2k, com k um nmero natural mpar. Vamos mostrar que, no
caso em que n = 2k, com k um nmero natural mpar, possvel colocar todas as fichas num mesmo
crculo
Numere os crculos consecutivamente, no sentido horrio, por: 1, 2, 3, 4, . . . , n.
Suponha que, em determinado instante, tenhamos ai fichas no crculo numerado com i e seja S a
expresso da soma das fichas nos crculos nesse instante. Isto :

S = 1 a1 + 2 a2 + 3 a3 + . . . + n an

Observe que, inicialmente ai = 1, para todo i = 1, 2, 3, . . . , n. Assim, inicialmente temos:


n(n + 1)
S = 11+21+31+...+n1 = .
2
Aps cada movimento, acontece o seguinte com o valor da expresso da soma das fichas em cada um dos n
crculos: ela no se altera ou ela se altera por n. Por exemplo, para n = 6 = 2 3. Inicialmente temos:
6(6 + 1)
S = 11+21+31+41+51+61 = = 21.
2
Se fizermos o movimento de colocar a ficha do crculo de nmero 2 para o crculo de posio 4 e passar a
ficha do crculo da posio 6 para o crculo da posio 4, a expresso S, que inicialmente 21, torna-se:

S = 1 1 + 2 0 + 3 1 + 4 3 + 5 1 + 6 0 = 1 + 3 + 12 + 5 = 21.

Por outro lado, fizermos o movimento de colocar a ficha do crculo de nmero 2 para o crculo de posio
4 e passar a ficha do crculo da posio 1 para o crculo da posio 5, a expresso S, que inicialmente
21, torna-se:
S = 1 0 + 2 0 + 3 1 + 4 2 + 5 2 + 6 1 = 3 + 8 + 10 + 6 = 27.
Assim, aps vrios movimentos a soma S torna-se
n(n + 1)
S= + q n, onde q Z.
2
Agora, quando todas as fichas encontram-se no crculo de nmero k, temos

S = 1 0 + 2 0 + 3 0 + . . . + (k 1) 0 + k n + (k + 1) 0 + . . . + n 0 = k n.
46 Captulo 2. ESTRATGIAS

Assim, temos

n(n + 1) n+1
+ q n = k n + q = k n + 1 = 2k 2q n = 2(k q) 1.
2 2

Portanto, n tem de ser um nmero mpar, que uma condio necessria para que o jogador possa
alcanar seu objetivo. fcil mostrar que esta condio tambm suficiente. De fato, podemos destacar
qualquer um dos crculo, digamos o crculo numerado com 1. Como n um nmero mpar, os restantes
n 1 crculos podem ser agrupados em pares. Cada par formado por dois crculo escolhidos de modo que
ambos estejam mesma distncia do crculo 1, sendo que colocados em direes opostas. Isto , os pares
so formados pelos crculos numerados por (i, n i + 2), para i = 2, 3, . . . , n. As fichas dos dois crculos
que formam um par podem ser levadas para o crculo numerado com 1 em um movimento. Portanto, neste
caso, o jogador pode colocar todas as fichas num mesmo crculo, formando uma pilha.

Exemplo-48:

O rei, pea do jogo de xadrez, se movimenta uma nica casa por vez em todas as direes. Pode o
rei, sem fazer movimentos diagonais, comeando da casa mais abaixo, esquerda, ir at a casa mais
acima direita, visitando cada uma das casas restantes do tabuleiro exatamente uma nica vez?

Soluo

Depois de ler o problema (se preciso for, mais de uma vez, talvez at em voz alta), de forma a saber
exatamente quais as quantidades dadas, quais as condies dadas (hipteses) e o que se pede, vamos traar
um plano para a soluo.

A resposta no.
Pinte as casas do tabuleiro alternadamente pretas e brancas. Agora, basta observar que o rei, a cada
movimento, sem fazer trajetos diagonais, vai de uma casa de uma cor para a casa de cor oposta. Como o rei
tem que fazer 63 desses movimentos para tingir a casa mais acima direita, o ltimo movimento ter que
deix-lo em uma casa da cor oposta a cor da primeira casa, pois 63 um nmero mpar . Mas, a primeira
casa e a ltima casa possuem a mesma cor, o que uma contradio. Portanto, o rei no pode, sem fazer
movimentos diagonais, comeando do quadrado mais abaixo, esquerda, ir at o quadrado mais acima
direita, visitando cada uma das casas restantes do tabuleiro exatamente uma nica vez.
Exemplo-49:

Numa sala quadrada de dimenses 13 13 retira-se o quadrado unitrio central. possvel


ladrilhar o restante da sala usando ladrilhos retangulares de dimenses 1 4 ou 4 1?

Soluo

Depois de ler o problema (se preciso for, mais de uma vez, talvez at em voz alta), de forma a saber
exatamente quais as quantidades dadas, quais as condies dadas (hipteses) e o que se pede, vamos traar
um plano para a soluo.

No possvel.
Pinte os quadrados do assoalho de preto e branco no padro seguinte. Na primeira linha acima, pinte de
preto os dois quadrados mais esquerda, pinte de branco os dois seguintes (da esquerda para direita), os
dois prximos pinte de preto, os dois prximos de branco, e assim por diante, de modo que na extremidade
direita fique um nico quadrado preto. Na segunda linha, pinte a linha de forma alternada com a pintura da
primeira linha: dois quadrados brancos, dois quadrados pretos e assim por diante. Numerando as linhas de
1 a 13, temos que todas as linhas mpar so pintadas de forma idntica que a pintura da primeira linha e
todas as linhas numeradas com nmeros pares so pintadas de forma idntica que a pintura da linha de
nmero 2, veja figura a seguir.
2.6 Faa um desenho e, dependendo da situao, pinte s cores. 47

Observe que existem mais quadrados unitrios pretos do que quadrados brancos. Por outro lado, cada
ladrilho 4 1, no importa como seja colocado, cobre precisamente dois quadrados unitrios pretos e dois
brancos. Assim, se um ladrilhamento deixar um nico quadrado descoberto, este quadrado tem de ser preto.
Mas, o quadrado central branco. Portanto, tal ladrilhamento impossvel.

Exemplo-50:

Uma pea L formada a partir de quadrados 2 2, dos quais retiramos um dos quatro
cantos, veja Figura A a seguir. Diga, justificando, se possvel cobrir um tabuleiro 5 7 ,
Figura B a seguir, usando peas L0 s, de modo que a cobertura, em vrias camadas, no cruze
as fronteiras do tabuleiro e de maneira tal que cada casa seja coberta pelo mesmo nmero de peas L.

Figura A- Pea L Figura B - Tabuleiro 5 7

Soluo
(Rssia - 1996). A resposta no.
Pintamos as casas do tabuleiro dado 5 7 alternadamente de preto e branco, de modo que as casas nos
quatro cantos sejam pretas. Em cada casa preta preta escrevemos o nmero 2, e em cada casa branca
escrevemos o nmero 1. Observe que a soma dos nmeros em todas as casas cobertas por cada pea L
no negativa, e consequentemente se cobrimos o tabuleiro em k nveis, a soma dos nmeros das casas
cobertas por aquelas peas L no negativa. Mas, se o nmero S e s a soma dos nmeros sobre o
tabuleiro, temos que

S = ks = k(2 12 + 23 1) = k < 0,

que uma contradio.


48 Captulo 2. ESTRATGIAS

Exemplo-51:

Em cada uma das 64 casas de um tabuleiro de xadrez 8 8 h um gro de acar. Uma formiga
comea em uma casa do canto do tabuleiro, come o acar, e se move para uma casa adjacente,
movimentando-se em direo horizontal ou vertical (mas nunca na diagonal). Continua deste modo
at acabar com todo o acar, e sem passar duas vezes por uma mesma casa. possvel que o
trajeto da formiga termine no canto do tabuleiro diagonalmente oposto ao inicial?

Soluo
Vamos demonstrar que no existe trajetria que satisfaa as condies exigidas.
Para isso, til pensar as casas do tabuleiro pintadas alternadamente de branco e preto. Agora, observe
que em cada movimento unitrio na direo horizontal ou vertical leva a formiga de uma casa para outra de
cor distinta da imediatamente anterior. Como o tabuleiro possui 8 8 = 64 casas, comeando em
qualquer uma delas so necessrios 63 movimentos para percorrer todas. Mas, fcil ver que depois
1, 3, 5 ou qualquer quantidade impar de movimentos estaremos em uma casa de cor diferente da inicial.
Portanto, a resposta ao problema negativa, pois a casa do canto diagonalmente oposto de mesma cor.
Exemplo-52:

Seja P um ponto na regio limitada pelo tringulo ABC. Traam-se por P as paralelas aos lados
do tringulo, que subdividem o tringulo em trs tringulos e trs paralelogramos. Se as reas dos
trs tringulos da subdiviso so, em alguma ordem, 9, 16 e 25, encontrar a rea do tringulo
ABC.

Soluo
Para ajudar na soluo, fazemos a seguir um desenho ilustrativo da situao.

A B

Para resolver o problema usaremos o seguinte fato: Se dois tringulos so semelhantes ento a razo entre
suas reas igual a razo entre lados homlogos.
Agora, observe que os trs tringulos da sub-diviso so semelhantes ao tringulo ABC, pois possuem
lados respectivamente paralelos.
Sejam S a rea do tringulo ABC e x, y, z os comprimentos dos lados paralelos ao lado AB dos
tringulos de rea 9, 16, 25, respectivamente.
As retas por P, paralelas aos lados BC e AC dividem o lado AB em trs segmentos de comprimentos
x, y, z (em alguma ordem). Assim, o comprimento do lado AB satisfaz:

AB = x + y + z.

Logo, temos
 x 2 9  y 2 16  z 2 25
= , = , = ,
x+y+z S x+y+z S x+y+z S
que equivalente a
x 3 y 4 z 5
= , = , = .
x+y+z S x+y+z S x+y+z S
2.6 Faa um desenho e, dependendo da situao, pinte s cores. 49

Somando membro a membro as trs igualdades acima, temos :


x y z 3 4 5 12 12
+ + = + + = 1 = S = 144.
x+y+z x+y+z x+y+z S S S S S
50 Captulo 2. ESTRATGIAS

2.7 Modifique o enunciado, para ver se lhe ocorre um caminho possvel


Quando o problema parecer difcil demais, modifique o enunciado, transforme o problema em outro, isto
pode levar a um caminho que lhe mostre a sada.

Exemplo-53:

Encontre todos os pares de inteiros positvos (x, y) para os quais temos

3 1 1
+ = e xy 3 6.
x y 2

Soluo

Depois de ler o problema (se preciso for, mais de uma vez, talvez at em voz alta), de forma a saber
exatamente quais as quantidades dadas, quais as condies dadas (hipteses) e o que se pede, vamos traar
um plano para a soluo.

3 1 1
Em vez de olhar para + = , vamos pensar numa equao equivalente a ela:
x y 2

3 1 1
+ = 2x + 6y = xy xy 2x 6y = 0 xy 2x 6y + 12 = 0 + 12 (x 6) (y 2) = 12.
x y 2

Assim, os possveis pares de inteiros positivos satisfazendo as condies acima so os elementos do


conjunto:
(18, 3), (12, 4), (10, 5), (9, 6), (8, 8), (7, 14).
Por outro lado, temos que os elemntos de cada par tem de satisfazer:

xy 3 6 xy 32 6 = 54.

Portanto, os pares de inteiros positvos (x, y) que queremos so: (18, 3), (9, 6), (8, 8), (7, 14).

Exemplo-54:

Calcule o valor da expresso E = (tan 15 ).(tan 30 ).(tan 45 ).(tan 60 ).(tan 75o ).

Soluo
A ideia aqui bservar que se x + y = 90 , ento
tan x + tan y
tan (x + y) = = tan 90 = .
1 tan x. tan y

Ou seja, 1 tan x . tan y = 0, que o mesmo que tan x . tan y = 1, ou ainda

1 1
tan x = = = cot (90o x )
tan y tan (90o x )

Logo, temos que


tan 75 = cot (90 75 ) = cot 15 ;
tan 60 = cot (90 60 ) = cot 30 .
Portanto,
E = (tan 15 ).(tan 30 ).(tan 45 ).(tan 60 ).(tan 75 ) =
= (tan 15 ).(cot 15 ).(tan 30 ).(cot 30 ).(tan 45 ) = 1.1.1 = 1.
2.7 Modifique o enunciado, para ver se lhe ocorre um caminho possvel 51

Exemplo-55:

Escreve-se em cada uma das seis faces de um cubo um nmero inteiro positivo. Para cada vrtice do
cubo, calcula-se o produto dos nmeros nas trs faces adjacentes a ele. A soma desses produtos
igual a 1001
Qual a soma dos seis nmeros escritos nas faces do cubo?

Soluo

Depois de ler o problema (se preciso for, mais de uma vez, talvez at em voz alta), de forma a saber
exatamente quais as quantidades dadas, quais as condies dadas (hipteses) e o que se pede, vamos traar
um plano para a soluo.

Em vez de tomar aleatoriamente os seis nmeros escritos nas faces do cubo, vamos considerar os nmeros
dois a dois, escritos nas faces opostas. Assim, sejam a1 , a2 , b1 , b2 , c1 , c2 os nmeros escritos nas faces dos
cubos de tal maneira que a1 , a2 esto escritos em faces opostas, b1 , b2 nmeros ecritos em faces opostas e
c1 , c2 nmeros escritos em faces opostas. Agora, a soma dos oito produtos:

a1 .a2 + a1 .b1 + a1 .b2 + a1 .c1 + + c1 .c2 = 1001.

Por outro lado,

a1 .a2 + a1 .b1 + a1 .b2 + a1 .c1 + + c1 .c2 = (a1 + a2 ).(b1 + b2 ).(c1 + c2 ) =

= 1001 = 7.11.13
. Portanto, a soma dos oito nmeros escritos nas faces do cubo igual a:

(a1 + a2 ) + (b1 + b2 ) + (c1 + c2 ) = 7 + 11 + 13 = 31.

Exemplo-56:

Pintam-se ou de vermelho ou de verde 2001 pontos distintos sobre um crculo. Em cada etapa,
todos os pontos so simultaneamente pintados novamente da maneira seguinte:
se dois pontos so vizinhos imediatos de um ponto P e possuem a mesma cor que P, ento a cor de
P no se altera, caso contrrio P muda de cor.
Comeando com a primeira pintura F1 , obtemos as pinturas F2 , f3 , , depois de aplicar vrias
etapas de pinturas.
Prove que existe um nmero n0 1000 tal que Fn0 = Fn0 +2 . Isto verdade se o nmero 1000 for
substitudo pelo nmero 999?

Soluo

Depois de ler o problema (se preciso for, mais de uma vez, talvez at em voz alta), de forma a saber
exatamente quais as quantidades dadas, quais as condies dadas (hipteses) e o que se pede, vamos traar
um plano para a soluo.
(BW 2001) A resposta no.
Vamos denotar os pontos sobre o crculo por 1, 2, 3, , 2001, de tal modo que os pontos i, j sejam vizinhos
se |i j| = 1 ou {i, j} = {1, 2001}.
Vamos dizer que k pontos formam um segmento monocromtico de comprimento k se os pontos so
consecutivos sobre o crculo e se eles possuem a mesma cor.
Para uma pintura F, chamamos de d(F) o comprimento mximo de um segmento monocromtico.
Observe que d(F) > 1, para todo n, pois 2001 mpar. Se d(F) = 2001, ento todos os pontos
possuem a mesma cor, portanto, F1 = F2 = F3 = e, neste caso, podemos escolher n0 = 1.
Seja 1 < d(F1 ) < 2001. Valem as seguintes propriedades:
(i) Se 3 < d(Fn ) < 2001, ento d(Fn+1 ) = d(Fn ) 2;
52 Captulo 2. ESTRATGIAS

(ii)Se d(Fn ) = 3, ento d( fn+1 ) = 2;


(iii) Se d(Fn ) = 2, ento d(Fn+1 = d(Fn e fn+2 = fn ;
Se valem as trs propriedades acima, segue de (i) e (ii) que d(F1000 ) 2 e por (iii) temos que
F1000 = F1002 . Alm disso, se F1 a pintura onde 1 pintado de vermelho e todos os outros pontos so
pintados de verde, ento d(F1 ) = 2000 e ento d(F1 ) > d(F2 ) > > d(F1000 ) = 2, o que mostra que
para todo n < 1000, temos Fn 6= Fn+2 e ento 1000 no pode ser substitudo por 999.
Agora, vamos provar as trs propriedades acima.
Seja (i + 1, i + 2, . . . , i + k) o maior segmento monocromtico para Fn , considerando os pontos mdulo
2001. Ento (i + 2, i + 3, . . . , i + k 1) um segmento monocromtico para Fn+1 e segue que
d(Fn+1 ) d(Fn 2. Alm disso, se (i + 1, i + 2, . . . , i + k) o maior segmento monocromtico para Fn+1 ,
com k 3, ento (i, i + 1, . . . , i + k + 1) um segmento monocromtico para Fn . Disso, e do fato de
Fn+1 > 1, seguem (i) e (ii).
Para provar (iii) observe que se d(Fn ) 2, ento Fn+1 obtido de Fn mudando as cores de todos os
pontos, o que conclui a prova.
2.8 Explore a simetria 53

2.8 Explore a simetria

Muitos problemas so resolvidos quando se explora a simetria que o problema apresenta, de forma explcita
ou mesmo mascarada.

Exemplo-57:

O professor de Matemtica de Jozinho apresenta um tabuleiro 6 6, veja Figura a seguir,

e prope o seguinte desafio: Joozinho tem de pintar de preto doze casas do tabuleiro, de modo
que em cada linha e em cada coluna apaream somente duas casas pintadas e, ao longo de uma
diagonal, no mais do que duas.
Joozinho vai conseguir resolver o desafio? Se sim, mostre como. Se no, explique porque.

Soluo

Depois de ler o problema (se preciso for, mais de uma vez, talvez at em voz alta), de forma a saber
exatamente quais as quantidades dadas, quais as condies dadas (hipteses) e o que se pede, vamos traar
um plano para a soluo.

Joozinho vai resolver o desafio. A resposta no nica. A estratgia de Joozinho consiste em dividir o
tabuleiro em quatro sub-tabuleiros 3 3, trabalhar num desses sub-tabuleiros e, em seguida, usar reflexo
em torno das retas que dividem o tabuleiro e torno de seu centro. A seguir, apresentamos duas solues.
54 Captulo 2. ESTRATGIAS

Exemplo-58:

Inscreve-se um quadrado num crculo, que por sua vez est inscrito noutro quadrado, veja figura a
seguir.

Qual a razo das reas dos dois quadrados?

Soluo
Podemos resolver o problema usando a noo de simetria. Se voc gira o quadrado menor por um ngulo de
45 , seus vrtices iro coincidir com os pontos de tangncia do crculo e do quadrado maior, veja figura a
seguir.

Portanto, fcil ver, que o quadrado menor possui a metade da rea do quadrado maior.

Exemplo-59:

Eve e Odette disputam um jogo num tabuleiro 3 3, com peas brancas e peas pretas. As regras
so as seguintes:
I - Elas jogam alternadamente;
II - Uma jogada consiste em colocar uma pea numa casa do tabuleiro ainda no ocupada
III - Na sua vez de jogar, uma jogadora pode escolher ou uma pea branca ou uma pea preta e no
precisa usar sempre a mesma cor;
IV - Quando o tabuleiro est totalmente preenchido pelas peas, Eves obtm um ponto para toda
linha, coluna ou diagonal que possuir um nmero par de peas pretas, e Odette obtm um ponto
para toda linha, coluna ou diagonal que possuir um nmero mpar de peas pretas;
V - A jogadora que obtiver no mnimo cinco dos oito pontos VENCE.
Descreva uma estratgia vencedora para a garota que comea o jogo.

Soluo
(CMO 1978)Vamos supor que Eve comea o jogo.
Eve coloca uma pea preta na casa central do tabuleiro, ela existe pois no tabuleiro temos 9 casas. A partir
da, sempre que Odette jogar, Eve joga com uma pea da cor oposta a de Odette na casa simtrica, em
relao a casa central, daquela que Odette jogou. Por este procedimento, Eve obtm um ponto para toda
linha, coluna e diagonal que passem pelo centro. Alm disso, pelo menos uma das linhas ou colunas que
passam pelo centro tem 2 peas pretas, o que implica que Eves vence o jogo.
Chamando de B uma pea branca e P uma pea preta, no final do jogo o tabuleiro ter a seguinte
aparncia:

P 1 B
P
P 2 B
2.8 Explore a simetria 55

Exatamente uma das casas 1 e 2 possui uma pea preta, e a linha dela possui um nmero par de peas
pretas.
56 Captulo 2. ESTRATGIAS

2.9 Reduo ao Absurdo.


Um raciocnio muito empregado na resoluo de problemas aquele que comumente se chama de mtodo
por reduo ao absurdo ou mtodo indireto.
Como que funciona?
Se voc pretende mostrar que uma afirmao A implica numa afirmao B, comece supondo que isto no
acontece. Ou seja, que A no implica em B. Fazendo dedues e raciocnios corretos, voc chegar
concluso de que uma afirmao, que voc sabe que correta, no se realiza. Ou seja, voc chegar a uma
contradio. Essa contradio provm do fato de termos assumido que A no implica em B. Portanto, A
implica em B.
Exemplo-60:

Demonstrar que o nmero 2 irracional.

Soluo
Depois de ler o problema (se preciso for, mais de uma vez, talvez at em voz alta), de forma a saber
exatamente quais as quantidades dadas, quais as condies dadas (hipteses) e o que se pede, vamos traar
um plano para a soluo.

Vamos provar por absurdo. Suponha o contrrio, isto , 2 no um nmero irracional. Isto significa
dizer que
m
2 = , onde m, n Z, com n 6= 0.
n
Sem perda de generalidade, vamos supor que os nmeros m e n sejam relativamente primos. Ou seja,
2
MDC(m, n) = 1. Elevando ao quadrado ambos os membros de 2 = mn , obtemos 2 = mn2 , que o mesmo
que dizer que
m2 = 2n2 ().
Assim, m2 um nmero par e, logo, m par. Desse modo, m = 2k, onde k Z. Substituindo esse valor
de m em (), obtemos que n2 par, o que implica que n par. Assim, conclumos
que m par e n
par. Uma contradio, pois tnhamos suposto MDC(m, n). = 1. Portanto, o nmero 2 irracional.
Exemplo-61:

Se a, b, c so nmeros inteiros mpares, prove que a equao quadrtica

ax2 + bx + c = 0 ()

no possui um nmero racional como soluo.

Soluo
Depois de ler o problema (se preciso for, mais de uma vez, talvez at em voz alta), de forma a saber
exatamente quais as quantidades dadas, quais as condies dadas (hipteses) e o que se pede, vamos traar
um plano para a soluo.
Vamos provar por absurdo. Suponha o contrrio, isto , existe um nmero racional qp com MDC(p, q) = 1,
que seja soluo da equao dada (). Assim, temos que
 p 2  p
a +b + c = 0 ap2 + bpq + cq2 = 0. ()
q q
Agora basta analisar a expresso (**) considerando a paridade de p e q:
Se p e q so ambos nmeros mpares.
Neste caso, a expresso ap2 + bpq + cq2 um nmero mpar, pois a, b, c so nmeros inteiros
mpares. Portanto, ap2 + bpq + cq2 6= 0, pois 0 um nmero par.
Se p um nmero par e q um nmero mpar.
Neste caso, temos que ap2 + bpq um nmero par, enquanto cq2 um nmero mpar. Logo,
ap2 + bpq + cq2 um nmero mpar. Portanto, no pode ser zero.
2.9 Reduo ao Absurdo. 57

Se p um nmero mpar e q um nmero par.


Neste caso, ap2 um nmero mpar, enquanto bpq + cq2 par. Logo, ap2 + bpq + cq2 um
nmero mpar. Portanto, no pode ser zero.
(Observe que que os nmeros p, q no podem ser ambos pares, pois MDC(p, q) = 1). Assim, se a equao
possui uma soluo racional, chegaremos a uma contradio. Portanto, a equao dada no possui um
nmero racional como soluo.
Exemplo-62:

Escrevem-se os nmeros 1, 2, 3, . . . , 49 nas casas de um tabuleiro 7 7, sendo um nmero em


cada casa e calcula-se a soma dos nmeros em cada linha e cada coluna. Algumas dessas 14 somas
so mpares enquanto as outras so pares. Seja A a soma de todas as somas mpares e B a soma
de todas as somas pares.
possvel colocar os nmeros nas casas do tabuleiro de modo que A = B?

Soluo

Depois de ler o problema (se preciso for, mais de uma vez, talvez at em voz alta), de forma a saber
exatamente quais as quantidades dadas, quais as condies dadas (hipteses) e o que se pede, vamos traar
um plano para a soluo.

A resposta no.
Se isto fosse possvel, teramos 2 (1 + 2 + 3 + 4 + . . . + 49) = A + B = 2B. Mas, B um nmero par,
pois a soma de nmeros pares. Por outro lado, 1 + 2 + 3 + 4 + + 49 = 25 49, que um nmero
mpar. Contradio. Portanto impossvel colocar os nmeros nas casas do tabuleiro de modo que A = B.
Exemplo-63:

Temos 2000 mas em vrias bolsas. Podemos remover bolsas e/ou remover mas das bolsas.
Prove que possvel ter um nmero igual de mas em cada uma das bolsas restantes, sendo o total
de mas no menor do que 100.

Soluo

Depois de ler o problema (se preciso for, mais de uma vez, talvez at em voz alta), de forma a saber
exatamente quais as quantidades dadas, quais as condies dadas (hipteses) e o que se pede, vamos traar
um plano para a soluo.

(TT - 1984) Suponha o contrrio. Assim, o nmero total de bolsas restantes no mais do que 99, caso
contrrio, poderamos deixar 1 ma em cada uma das 100 bolsas e remover as restantes. Alm disso, o
total de bolsas com no mnimo duas mas no mais do que 49, o total de bolsas com no mnimo trs
mas no mais do que 33, etc. Portanto, o total de mas no maior do que 99 + 49 + 33 + . . ..
Existem 94 termos depois dos cinco primeiros termos , cada um deles sendo menor do que ou igual a 16.
Agora
99 + 49 + 33 + 24 + 19 + 94 16 = 1728 < 2000.
Contradio. Portanto, possvel ter um nmero igual de mas em cada uma das bolsas restantes, sendo o
total de mas no menor do que 100.
Exemplo-64:

Prove que no existem nmeros inteiros positivos m e n para os quais

m2 n2 = 1. ()

Soluo

Depois de ler o problema (se preciso for, mais de uma vez, talvez at em voz alta), de forma a saber
exatamente quais as quantidades dadas, quais as condies dadas (hipteses) e o que se pede, vamos traar
um plano para a soluo.
58 Captulo 2. ESTRATGIAS

Suponha o contrrio. Isto , suponha que existam nmeros inteiros positivos m e n para os quais
m2 n2 = 1. Assim, temos

m2 n2 = 1 (m + n) (m n) = 1 ()

Como m e n so inteiros positivos, segue que m + n e m n so nmeros inteiros. Logo, de ()


conclumos que
m+n = 1 e mn = 1 ( )
ou
m + n = 1 e m n = 1 ( )
Agora, somando membro a membro as duas equaes de ( ), obtemos 2m = 2 m = 1 n = 0.
Contradio, pois n um nmero inteiro positivo.
De mdulo anlogo, somando membro a membro as duas equaes de ( ), obtemos 2m = 0 m = 0.
Contradio, pois m um nmero inteiro positivo.
Portanto, no existem inteiros positivos m e n para os quais m2 n2 = 1.
Exemplo-65:

Seja p um nmero primo, p > 3. Sabe-se que para um certo inteiro positivo n o nmero pn possui
20 dgitos, quando escrito na base 10.
Prove que dentre esses dgitos existem no mnimo trs iguais.

Soluo

Depois de ler o problema (se preciso for, mais de uma vez, talvez at em voz alta), de forma a saber
exatamente quais as quantidades dadas, quais as condies dadas (hipteses) e o que se pede, vamos traar
um plano para a soluo.

(KMO- 1981)Suponha o contrrio. Isto , dentre os 20 dgitos do nmero pn no existem trs iguais. Isto
significa que um dos dgitos 0, 1, 2, 3 , 9 usado na representao decimal de pn exatamente duas
vezes. Assim, a soma dos dgitos de pn igual a

2 (0 + 1 + 2 + + 9) = 90.

Logo, pn divisvel por 3. Mas, isso uma contradio, pois p um primo maior do que 3, o que
implica que pn no pode ser divisvel por 3. Portanto, dentre os dgitos de pn existem no mnimo trs
iguais.
Exemplo-66:

Prove que o polinmio P(x) = x5 x2 + 2x 1, com coeficientes inteiros, no admite razes


negativas.

Soluo

Depois de ler o problema (se preciso for, mais de uma vez, talvez at em voz alta), de forma a saber
exatamente quais as quantidades dadas, quais as condies dadas (hipteses) e o que se pede, vamos traar
um plano para a soluo.

Suponha por absurdo que o polinmio P(x) dado admite uma raiz negativa. Seja r < 0 esta raiz. Assim,
temos:
P(r) = r5 r2 + 2r 1 = 0 r5 = r2 2r + 1 = (r 1)2 . ()

Agora, observe que o lado direito da igualdade () positivo, enquanto o lado esquerdo negativo.
Contradio, que decorre do fato de termos assumido que o polinmio dado admitia uma raiz negativa.
Portanto, o polinmio P(x) = x5 x2 + 2x 1, com coeficientes inteiros, no admite razes negativas.
2.9 Reduo ao Absurdo. 59

Exemplo-67:

Um tabuleiro 6 6 pode ser coberto por 18 domins 2 1, de modo que cada domin cubra
perfeitamente duas casas do tabuleiro.
Prove que, no importa como fazemos a cobertura (dentro das hipteses), podemos traar uma reta
paralela a um dos bordos dividindo a cobertura em duas partes e tal que nenhum domin fique
cortado ao meio por esta reta.

Soluo

Depois de ler o problema (se preciso for, mais de uma vez, talvez at em voz alta), de forma a saber
exatamente quais as quantidades dadas, quais as condies dadas (hipteses) e o que se pede, vamos traar
um plano para a soluo.

Observe que, como o tabuleiro 6 6, existem 5 retas horizontais e 5 retas verticais dividindo o
tabuleiro, veja figura a seguir.

No importa qual seja a cobertura feita, cada uma destas retas corta um nmero par de domins. De fato,
se uma dessas retas cortar um nmero mpar de domins, o tabuleiro ficar dividido em duas partes, cada
uma delas conter um nmero mpar de metades de um domin. Mas, isto impossvel, pois cada uma
dessas partes possui um nmero inteiro de casas cobertas por domins e existem 36 casas no tabuleiro
todo. Portanto, toda reta (paralela a um dos bordo do tabuleiro) corta um nmero par de domins, de modo
que os arranjos dos 18 domins sobre as dez linhas sejam tais que no mnimo uma reta no corta ao meio
qualquer domin.
60 Captulo 2. ESTRATGIAS

2.10 Suponha o problema resolvido (ou olhe de trs para diante)


Uma ttica que pode ser utilizada em jogos ou problemas em que se tem de construir alguma figura, supor
o problema resolvido. Ento voc poder reverter seus passos e, portanto, construir uma soluo para o
problema original.
Exemplo-68:

Dois amigos, A e B se divertem com o seguinte jogo, em que jogam alternadamente. O primeiro a
jogar, A, escolhe um inteiro qualquer de 1 a 11, inclusive, e comunica ao segundo jogador, B, a
sua escolha e este soma-o qualquer nmero de 1 a 11, inclusive, por ele escolhido, falando o
resultado ao primeiro jogador que, na sua vez de jogar, o soma qualquer nmero escolhido de 1
a 11, e assim eles vo jogando, alternadamente, at que um deles obtenha o nmero 56, vencendo
o jogo.
Quem vence: A ou B? Qual a estratgia para vencer?

Soluo

Depois de ler o problema (se preciso for, mais de uma vez, talvez at em voz alta), de forma a saber
exatamente quais as quantidades dadas, quais as condies dadas (hipteses) e o que se pede, vamos traar
um plano para a soluo.

Depois de ler o problema (se preciso for, mais de uma vez, talvez at em voz alta), de forma a saber
exatamente quais as quantidades dadas, quais as condies dadas (hipteses) e o que se pede, vamos traar
um plano para a soluo.

O jogador A, que comea, vence.


A estratgia do jogador A olhar para a situao no final do jogo. Assim, na sua penltima jogada, o
jogador A tem de deixar para B uma soma menor do que 56 11 = 45, para garantir que B no possa
atingir o nmero 56. Assim, ele deixa a soma igual a 44. Na sua jogada imediatamente anterior ele deixa
para o jogador B uma soma menor do que 44 11 = 33, para garantir que o jogador B no possa
atingir 44. Desse modo, as jogadas vencedoras de A so de tal maneira que os totais atingidos por eles
sejam: 8, 20, 32, 44 e 56.
Exemplo-69:

Dois amigos, A e B, disputam seguinte jogo, em que jogam alternadamente, usando um monte com
27 de caroos de feijo. O jogador A comea. Uma jogada consiste em retirar 1, 2, 3 ou 4
caroos do monte. O jogo termina quando todos caroos forem removidos. O vencedor aquele que
possuir um nmero par de caroos quando todos os caroos tiverem sido retirados.
Quem vence: A ou B? Qual a estratgia vencedora?

Soluo

Depois de ler o problema (se preciso for, mais de uma vez, talvez at em voz alta), de forma a saber
exatamente quais as quantidades dadas, quais as condies dadas (hipteses) e o que se pede, vamos traar
um plano para a soluo.

(Soluo por David Angell, University of New South Wales, Australia. Fonte: ANGELL, D]
Descreva a posio do jogo num momento qualquer por um nmero n par:
(n, I), se existem no monte n caroos de feijo e se o jogador que vai fazer seu movimento possui um
nmero mpar de caroos ou
(n, P) se existem no monte n caroos de feijo e se o jogador que vai fazer seu movimento possui um
nmero par de caroos.
Dizemos que uma posio uma posio vencedora se o jogador que tem a vez de jogar pode garantir que
vai vencer, no importando como o outro jogue. Assim, temos:
(1, I) uma posio vencedora, porque o jogador que vai fazer seu movimento retira o ltimo
caroo e fica com um nmero par deles.
2.10 Suponha o problema resolvido (ou olhe de trs para diante) 61

(1, P) uma posio perdedora, pois o jogador que vai fazer seu movimento retira o ltimo caroo e
fica com um nmero mpar deles.
(2, I) uma posio vencedora, porque retirando um caroo fica com um nmero par deles e deixa o
oponente na posio (1, P), que uma posio perdedora..
(2, P) uma posio vencedora, pois o jogador que vai fazer seu movimento retira os dois ltimos
caroos e termina com um nmero par deles.
Por razes anlogas, (3, I), (3, P), (4, I) e (4, P) so posies vencedoras. Por exemplo, na posio
(3, P) o oponente possui um nmero par de caroos, retirando dois caroos coloca o oponente na
posio (1, P), que, como vimos acima, uma posio perdedora.
(5, I) uma posio perdedora, pois o oponente possui um nmero mpar de caroos, pois qualquer
movimento deixa-o em uma das posies (4, I) ou (3, I) ou (2, I) ou (1, I).
(5, P) uma posio vencedora, porque retirando 4 caroos coloca o oponente na posio
perdedora (1, P).
(6, I) uma posio perdedora porque o oponente possui um nmero par de caroos, e qualquer
movimento leva-o para uma das posies vencedoras (5, P) ou (4, P) ou (3, P) ou (2, P).
(6, P) uma posio vencedora, pois retirando 1 caroo coloca o oponente na posio perdedora
(5, I). Se voc continuar com esse raciocnio, vai descobrir que agora a situao se repete: as
posies vencedoras e perdedoras para n caroos so as mesmas que para n 6 caroos. Por
exemplo, se n = 7, ento (n, I) vence e (n, P) perde, mesmo para n = 1; se n = 8, ento ambas
as posies (n, I) e (n, P) vencem; o mesmo para n = 2, e assim por diante.
Portanto, para n = 27, comeando com um nmero par de caroos (0 um nmero par) essencialmente
o mesmo que a posio (3, P) e o primeiro jogador vence retirando dois caroos.
A estratgia vencedora simplesmente seguir a tabela de instrues dada abaixo. Como existem realmente
somente doze posies de (1, I) a (6, P), e trs delas so posies perdedoras, segue que no importa
como voc jogue, existem nove posies vencedoras para memorizar. Essas posies vencedoras esto na
tabela abaixo, onde n (mod 6) significa o resto quando n dividido por 6.

n (mod 6) 0 1 2 3 4 5
n Impar nenhum 1(2) 1 3(4) 3 nenhum
n Par 1 nenhum 2(3) 2 4 4
62 Captulo 2. ESTRATGIAS

2.11 O Princpio da Induo.


Em muitos problemas da Matemtica precisamos verificar a veracidade de uma afirmao, A(n), que
depende de um nmero natural n. Para fazer a verificao, usamos o mtodo de induo.
Em que consiste o mtodo de induo matemtica?
A prova por induo pode ser pensada como a brincadeira de arrumar domins em fila e derrub-los como
uma onda: derrubamos a primeira pea, que ao cair bate na segunda derrubando-a, que ao cair bate na
terceira derrubando-a e assim por diante, at que todas elas sejam derrubadas, conforme mostra a Figura 1
a seguir.

80
81
76777879
82
8483
86 85

737475
87
88

72
71
70
69
68
6766
6564
63 6261
60
59
58
57
56
5 55
5 53 4
5 51 2
49 0
48
47
46
45
4443
4241 0
4 39 8
337
36
35
33 34
332
2 2930 1
8
26 27
25
24
23
22
2210
1918
1716 5
1 143
1 2
111
10
9
7 8
56
4
3
2
1

Figura 1 - Domins em queda - Fonte:http://tex.stackexchange.com/a/151474

Agora, em vez de peas de domins, pense numa seqncia de afirmaes: A(1), A(2), A(3), . . . , A(n), .
Imagine que seja possvel provar as duas etapas seguintes:

1. a primeira afirmao, A(1), verdadeira;


2. sempre que uma afirmao for verdadeira, a imediatamente seguinte tambm verdadeira.

Conclumos que todas as afirmaes A(1), A(2), A(3), . . . , A(n), so verdadeiras.

Relacionando com as peas de domin,


1. seria a derrubada da primeira pea,
2. seria a queda de uma pea de domin provocada pela queda da pea anterior.

A parte 1. chamada a hiptese de induo ou a base da induo e a 2. a etapa indutiva.

z Observao. Veja que natural aceitarmos o mtodo de induo como um princpio, pois o que
ele garante a cerca de uma afirmao A(n) que: A(1) verdadeira = A(2) verdadeira = A(3)
verdadeira = A(4) verdadeira = . . . = A(n) verdadeira = A(n+1) verdadeira = . . .. Ou
seja, essa sequncia de implicaes varre todos os nmeros naturais.
2.11 O Princpio da Induo. 63

Exemplo-70:

(A soma dos primeiros n nmeros naturais)


Provar que, para todo nmero natural n, temos:

n(n + 1)
1+2+3+...+n =
2

Soluo

Observe que a afirmao A(n) a ser provada : 1 + 2 + 3 + . . . + n = n(n+1)


2 , para cada nmero natural
n.
Assim, A(1) ser entendida como a afirmao: 1 = 1(1+1) 2 ;
2(2+1)
A(2) : 1 + 2 = 2 ;
3(3+1)
A(3) : 1 + 2 + 3 = 2 , e assim por diante.
Etapa 1 Vamos verificar a base da induo. Isto , (i), que o mesmo que verificar que A(1)
verdadeira. Para isso, basta observar que: 1 = 1(1+1)
2 .

Etapa 2 Vamos supor que para n = k, onde k um nmero natural maior do que ou igual a 1,
a frmula dada seja verdadeira. Isto , 1 + 2 + 3 + . . . + k = k(k+1)
2 . Agora, provaremos que para
n = k + 1 a igualdade tambm se verifica. De fato, somando k + 1 em ambos os membros da
expresso anterior, obtm-se:
k(k + 1) k(k + 1) + 2(k + 1) (k + 1) (k + 2)
1 + 2 + 3 + . . . + k + (k + 1) = + (k + 1) = = .
2 2 2
Esse valor coincide com o valor da frmula dada ao substituirmos n por k + 1, pois:
(k + 1).[(k + 1) + 1] (k + 1)(k + 2)
1 + 2 + 3 + . . . + k + (k + 1) = = .
2 2
Portanto, pelo Princpio da Induo, completamos a prova.

Exemplo-71:

(A soma dos quadrados dos primeiros n nmeros naturais)


Provar que, para todo nmero natural n, temos:

n(n + 1)(2n + 1)
12 + 22 + 32 + . . . + n2 =
6

Soluo

Observe que a afirmao A(n) a ser provada : 1 + 2 + 3 + + n = n(n+1)


2 , para cada nmero natural
n.
Assim, A(1) ser entendida como a afirmao: 1 = 1(1+1) 2 ;
2(2+1)
A(2) : 1 + 2 = 2 ;
3(3+1)
A(3) : 1 + 2 + 3 = 2 , e assim por diante.
Etapa 1 Vamos verificar a base da induo. Isto , (i), que o mesmo que verificar que A(1)
verdadeira. Para isso, basta observar que: 1 = 1(1+1)
2 .

Etapa 2 Vamos supor que para n = k, onde k um nmero natural maior do que ou igual a 1,
a frmula dada seja verdadeira. Isto , 1 + 2 + 3 + + k = k(k+1)
2 . Agora, provaremos que para
n = k + 1 a igualdade tambm se verifica. De fato, somando k + 1 em ambos os membros da
expresso anterior, obtm-se:
k(k + 1) k(k + 1) + 2(k + 1) (k + 1) (k + 2)
1 + 2 + 3 + . . . + k + (k + 1) = + (k + 1) = = .
2 2 2
64 Captulo 2. ESTRATGIAS

Esse valor coincide com o valor da frmula dada ao substituirmos n por k + 1, pois:

(k + 1).[(k + 1) + 1] (k + 1)(k + 2)
1 + 2 + 3 + . . . + k + (k + 1) = = .
2 2
Portanto, pelo Princpio da Induo, completamos a prova.

Exemplo-72:

(A soma dos quadrados dos primeiros n nmeros naturais)


Provar que, para todo nmero natural n, temos:

n(n + 1)(2n + 1)
12 + 22 + 32 + . . . + n2 =
6

Soluo

A afirmao, A(n), a ser provada : 12 + 22 + 32 + . . . + n2 = n(n+1)(2n+1)


6 , para todo nmero natural
n. Assim,
A(1) ser entendida como a afirmao: 1(1+1)(21+1)
6 = 23
6 =1=1 ;
2
2(2+1)(22+1)
A(2) : 12 + 22 = 6 ;
A(3) : 12 + 22 + 32 = 3(3+1)(23+1)
6 , e assim por diante.

Etapa 1 Vamos verificar a base da induo, isto , (i). Para isso, basta observar que: 12 =
1(1+1)(21+1)
6 = 23
6 = 1.

Etapa 2 Vamos supor que, para n = k, onde k um nmero natural maior do que ou igual a 1, a
frmula dada seja verdadeira. Isto , 12 + 22 + 32 + . . . + k2 = k(k+1)(2k+1)
6 para todo nmero natural
k.
Agora, provaremos que para n = k + 1 a igualdade tambm se verifica. De fato, somando (k + 1)2
em ambos os membros da igualdade anterior, obtm-se:

k(k + 1)(2k + 1) k(2k + 1)


12 + 22 + 32 + . . . + k2 + (k + 1)2 = + (k + 1)2 = (k + 1) + (k + 1)2 =
6 6

2k2 + k + 6k + 6 2(k + 32 )(k + 2)


= (k + 1) = (k + 1) =
6 6
(k + 1)(2k + 3)(k + 2) (k + 1)(k + 2)[2(k + 1) + 1]
= = ,
6 6
que a expresso n(n+1)(2n+1)
6 quando n = k + 1, o que completa a prova. Observe que na anti-
penltima igualdade acima foi usado que o trinmio do segundo grau 2k2 + 7k + 6 se fatora em
2(k + 2)(k + 32 ).

Exemplo-73:

(A soma dos cubos dos primeiros n nmeros naturais)


Provar que, para todo nmero natural n, temos:
 2
3 3 3 n(n + 1)
3
1 +2 +3 +...+n =
2

Soluo
2.11 O Princpio da Induo. 65

A afirmao, A(n), a ser provada : 13 + 23 + 33 + . . . + n3 = [ n(n+1) ]2 , para todo nmero natural


h i2 2
n. Assim, A(1) ser entendida como a afirmao: 1(1+1) 2 = 22 = 1 = 13 ; A(2) : 13 + 23 = 9 =
h i2 h i2
2(2+1)
2 ; A(3) : 13 + 23 + 33 = 36 = 3(3+1)
2 , e assim por diante.
Etapa 1 Vamos verificar a base da induo. Isto , vamos verificar que (i) verdadeira. Para isso,
basta observar que:
h 1(1 + 1) i2  2 2
= = 12 = 1 = 13 .
2 2
Etapa 2 Vamos supor que para n = k, onde k um nmero natural maior do que ou igual a 1, a
frmula dada seja verdadeira. Isto , 13 + 23 + 33 + + k3 = [ k(k+1) 2
2 ] , para todo nmero natural k.
Agora, provaremos que para n = k + 1 a igualdade tambm se verifica. De fato, adicionando (k + 1)3
a ambos os membros, temos:
 2
3 3 3 3 3k(k + 1)
1 + 2 + 3 + + k + (k + 1) = + (k + 1)3 =
2

(k + 1)2 k2 + 4(k + 1)
 
k2 (k + 1)2 3 k2 (k + 1)2 + 4(k + 1)3
+ (k + 1) = = =
4 4 4
(k + 1)2 [k + 2]2 (k + 1)(k + 2) 2
 
= = ,
4 2
que a frmula para n = k + 1, o que completa a prova.

zNota - Observe que, pelo exemplo 1.3.1, temos que 1 + 2 + 3 + + n =


exemplo 1.3.3, curiosamente, nos permite concluir que:
n(n+1)
2 . Agora, o

 2
3 3 3 3 2 n(n + 1)
1 + 2 + 3 + . . . + n = (1 + 2 + 3 + . . . + n) = ,
2

um resultado, convenhamos, surpreendente!

Exemplo-74:

Tm-se 2n moedas de ouro, sendo uma delas falsa, com peso menor que as demais. Dispe-se
de uma balana de dois pratos, sem nenhum peso. Mostre, por induo, que possvel achar a
moeda falsa com n pesagens.

Soluo

Para n = 1 teremos 2n = 21 = 2 moedas. Neste caso colocamos uma moeda em cada prato da
balana. Aquele prato que ficar mais alto conter a moeda mais leve e, portanto, com apenas uma
pesagem conseguimos, nesse caso, descobrir a moeda falsa.
Suponhamos que a proposio seja verdadeira para o nmero natural n = k, isto , com 2k moedas,
uma quantidade de k pesagens so suficientes para descobrirmos a moeda falsa (Hiptese da induo).
Agora, vamos mostrar que a proposio vlida para n = k + 1, ou seja, com 2k+1 moedas podemos
descobrir a moeda falsa com k + 1 pesagens. Com efeito, se possuirmos 2k+1 moedas podemos
dividir todas essas moedas em dois grupos com 2k moedas cada um, pois 2k+1 = 2.2k . Colocamos
ento cada um desses dois grupos nos pratos da balana. Dessa forma, um dos pratos da balana ir
ficar mais alto (pois a moeda falsa estra num desses dois grupos e portanto h um grupo mais leve que
outro). O prato da balana que ficar mais alto acusar em qual dos grupos estar a moeda falsa. Note
que agora temos um grupo com 2k moedas, onde necessariamente est a moeda falsa. Pela hiptese
da induo, com k pesagem podemos descobrir qual a moeda falsa. Assim, com um grupo de 2k+1
moedas, onde apenas uma falsa (mais leve), k + 1 pesagem so suficientes para descobrir qual a
66 Captulo 2. ESTRATGIAS

moeda falsa; uma pesagem para descobrir qual o grupo em que est a moeda falsa e k pesagens para
descobir qual , de fato, a moeda falta, resultanto ento em um total de k + 1 pesagens.
Exemplo-75:

Um grupo de n 2 pessoas est em uma fila para comprar entradas para o cinema. A primeira
pessoa da fila uma mulher e a ltima um homem. Mostre que, em algum ponto da fila, uma
mulher est diretamente na frente de um homem.

Soluo

Para n = 2, teremos uma fila com apenas duas pessoas, na qual a primeira pessoa da fila uma mulher
e a segunda um homem, o que revela que h na fila uma mulher diretamente frente de um homem.
Suponhamos que a proposio seja verdadeira para o nmero natural n = k, ou seja, numa fila com k
pessoas em que a primeira pessoa uma mulher e a ltima um homem em algum ponto da fila,uma
mulher est diretamente na frente de um homem (hiptese da induo).
Agora vamos demonstrar que a proposio verdadeira para n = k + 1, ou seja, que numa fila com
k + 1 pessoas em que a primeira pessoa uma mulher e a ltima um homem em algum ponto da
fila,uma mulher est diretamente na frente de um homem. De fato, seja

a1 a2 a3 . . . ak ak+1

uma fila com k + 1 pessoas em que a primeira pessoa da fila uma mulher e a ltima um homem.
Se a pessoa a2 for um homem a proposio foca demonstrada, pois a1 uma mulher e a teremos
ento uma mulher diretamente frente de de um homem. Se a2 for uma mulher, teremos ento uma
fila com k pessoas, a2 a3 ak ak+1 onde a primeira pessoa da fila uma uma mulher e a ltima
um homem, que pela hiptese da induo, satisfaz a proposio. Assim, de qualquer forma temos o
resultado desejado, ou seja, que numa fila com k + 1 pessoas, em que a primeira pessoa uma mulher
e a ltima um homem em algum ponto da fila,uma mulher est diretamente na frente de um homem.
Exemplo-76:

Se x + 1x um nmero inteiro, mostre que xn + x1n nmero inteiro para todo n N.

Soluo

Para n = 1 a proposio claramente verdadeira, pois para n = 1 a expresso xn + x1n transforma-se


em x + 1x , que por hiptese, um nmero inteiro.
Usando o princpio da induo segunda forma, suponhamos que a expresso xn + x1n , seja um nmero
inteiro para n = 2, 3, , k (hiptese da induo).
Agora vamos provar que para n = k + 1 a expresso xn + x1n um nmero inteiro. Com efeito, temos
que   
1 1 1 1
x+ x + k = xk+1 + k+1 + xk1 + k1
k
x x x x
    
k+1 1 1 k 1 k1 1
x + k+1 = x + x + k x + k1
x x x x
ora, como x + 1x , por hiptese inteiro, e xn + x1n , um nmero inteiro para n = 2, 3, , k, segue que
no segunto membro da igualdade
    
k+1 1 1 k 1 k1 1
x + k+1 = x + x + k x + k1
x x x x
1
todos os nmeros so inteiros, o que revela que a expresso xk+1 + xk+1 um nmero inteiro, o que
demonstra o resultado desejado.
3. Problemas Diversos

Exemplo-77:

Comeando com um hexgono regular, uma forma geomtrica gerada em estgios. Na Fase
1, h um hexgono. Cercando-o completamente, adicionando um hexgono regular congruente
ao incial, para cada lado dele exposto, temos que na Fase 2, existem 7 hexgonos no total.
Cercando completamente a nova forma pela adio de um hexgono regular, congruentes aos
outros, para cada lado exposto, temos que na Fase 3 existem um total de 19 hexgonos, veja
Figura a seguir.

Encontre uma frmula para o nmero total de hexgono em Fase n.

Soluo
(AMO) No hexagono inicial, traamos as trs maiores diagonais e as prolongamos. Elas cortam a
figura em 6 partes congruentes, veja Figura a seguir.
68 Captulo 3. Problemas Diversos

Agora, omitindo o hexgono central, observa-se que a quantidade de hexgono em cada parte
congruente e em cada estgio :
     
1 1 1 1 1 1
1, +1+ , 3, +3+ , 5, +5+ , 7, . . .
2 2 2 2 2 2

Isto , a quantidade de hexgonos em cada parte congruente e em cada estgio :

1, 2, 3, 4, 5, 6, 7, . . . , n 1 (depois de n est agios)


Portanto, se Tn o nmero de hexgonos no estgio n, temos que

6n(n 1)
Tn = 6[1 + 2 + 3 + 4 + 5 + + (n 1)] + 1 = + 1 = 3n(n 1) + 1 = 3n2 3n + 1.
2

Exemplo-78:

Sejam m e n nmeros inteiros positivos e um tabuleiro m n, onde cada casa corresponde a


um quadrado 1 1. No tabuleiro, existem quantos retngulos distintos, cada um deles tendo
os lados paralelos aos bordos?

Soluo

Observe que cada retngulo definido por duas retas horizontais e duas retas verticais. Por outro lado,
no tabuleiro dado existem m + 1 retas horizontais e n + 1 retas verticais, veja a Figura a seguir para
o caso em que m = 7 e n = 5.

Para formar um retngulo nas condies do problema, temos que escolher duas retas horizontais e duas
retas verticais. Agora, observe que as retas retas horizontais so escolhidas de forma independente
das retas verticais e a ordem pelas quais se faz a escolha das duas retas horizontais e das duas retas
verticais so irrelevantes. Ainda, no so permitidas a escolha de retas repetidas, pois, caso contrrio,
69

teramos algum retngulo com rea zero.


A escolha de duas retas horizontais distintas dentre as dentre m + 1 existentes se d de m+1

2 ea
escolha de duas retas verticais distintas dentre as dentre n + 1 existentes se d de n+1

2 . Pelo Princpio
Multiplicativo da Anlise Combinatria, a resposta m+1 n+1

2 2 .

Exemplo-79:

Num tabuleiro de xadrez (8 8), quantos cavalos (pea do jogo de xadrez) podemos arrumar
de modo que dois quaisquer deles no se ameacem mutuamente?

Soluo

Inicialmente, observe que, no jogo de xadrez, o cavalo, que a pea mais "gil", se movimenta em
forma de L: de uma casa sobre a coluna em que se encontra para a casa localizada contando-se duas
casas (para cima ou para baixo) na mesma coluna e uma casa (para a esquerda ou para direita) na linha
perpendicular (ou contando-se duas casas (para esquerda ou para direita) na mesma linha e uma casa
(para cima ou para baixo) na coluna perpendicular. Isto , o cavalo se movimenta de um canto para
o canto oposto de um retngulo 2 3 ou 3 2. Por exemplo, se o cavalo est na casa d4 (isto
, a casa que se encontra na interseo da coluna d com a linha 4) em um movimento, existem 8
possveis casas para ele ocupar. Observe que o cavalo se movimenta no espao!, veja Figura AP8 a
seguir.

Figura AP8- Movimentos do cavalo

Observe tambm que um movimento de um cavalo se d sempre de uma casa para outra de cor oposta,
veja Figura a seguir.

Figura - Em um movimento, o cavalo se desloca para uma casa de cor oposta


70 Captulo 3. Problemas Diversos

Isto significa que, em um nico movimento, o cavalo sai de uma casa de cor branca para uma casa de
cor preta, e vice versa.
Logo, se o cavalo est numa casa branca ele s pode ameaar peas que estejam em casas de cor
preta. Deste modo, como o tabuleiro possui 32 casas brancas, podemos colocar 32 peas sem que se
ameacem mutuamente. Por outro lado, se colocamos 33 cavalos, um deles estar obrigatoriamente
numa casa preta e, neste caso, existir um cavalo, localizado numa casa branca que o ameaar.
Portanto, a resposta 32.

Exemplo-80:

Juliette e Philippe disputam um jogo em que fazem seus movimentos alternadamente. No incio
do jogo, cada vrtice de um quadrado coberto por um nmero de fichas. Um movimento
consiste em cada jogador escolher um lado do quadrado e retirar de suas extremidades tantas
fichas quanto ele desejar, desde que ele retire pelo menos uma.
A B


D C
No necessrio retirar o mesmo nmero de fichas de cada extremidade. O jogador que retirar
a ltima ficha vence o jogo. No incio do jogo, existem 10 fichas no vrtice A, 11 fichas no
vrtice B, 12 fichas no vrtice C e 13 fichas no vrtice D.
Se Juliette comea, qual a sua estratgia para vencer?

Soluo
(CM, 37: No 5 September 2011) Sejam a, b, c e d o nmero de fichas nos vrtices A, B,C e D,
respectivamente. A estratgia de Juliette fazer seus movimentos para ter a = c e b = d.
Se na sua vez de jogar, Philippe recebe um quadrado com a = c e b = d, ento ele retira no mnimo
uma ficha, e no pode retirar fichas de ambos os extremos de uma diagonal. Portanto, ele sempre passa
para Juliette um quadrado com a 6= c e b 6= d. Se Juliette recebe um quadrado a 6= c e b 6= d, ento
para cada diagonal ela deve escolher a extremidade com maior nmero de fichas. Nesse caso, ela deve
escolher o lado que liga essas duas extremidades e retirar fichas at que a = c e b = d. Assim, ela
sempre pode aplicar essa estratgia e obter a = c = 0 e b = d, vencendo o jogo.

Exemplo-81:

Tem-se cartes sobre uma mesa de modo que em cada um deles est escrito um par de nmeros
inteiros positivos. Existe exatamente um carto para cada par de nmeros inteiros, (a, b),
com 1 a < b 2003. Dois jogadores disputam um jogo, em que jogam alternadamente.
Cada movimento corresponde a retirar um carto e escrever no quadro-negro o produto ab
dos nmeros l escritos. O primeiro jogador que faz com que o Mximo Divisor Comum
dos nmeros no quadro-negro seja igual a 1 perde.
Qual dos jogadores tem uma estratgia vencedora: o primeiro ou o segundo a jogar?

Soluo
(OMM - 2003) O primeiro jogador tem uma estratgia vencedora.
Considere os nmeros no quadro-negro imediatamente antes do movimento perdedor. O Mximo
Divisor Comum de todos esses nmeros deve ser um inteiro d > 1.
71

Se d no um nmero primo, ento ele possui algum divisor, k, menor do que ele, de modo que o
carto com o par (1, k) ainda pode ser retirado. Contradio, pois o prximo movimento perdedor.
Logo, d um nmero primo e todo carto com um par (a, b), com d dividindo ab deve ter j
sido retirado. Existem 2002 pares da forma (a, d), pois, pela hiptese do problema, a pode ser
qualquer um dos nmeros 1, 2, 3, . . . 2003 exceto d. De maneira anloga, existem 2002 pares da
forma (a, 2d), com 2d 2003. Agora, observe que, deste modo, contamos duas vez o par (d, 2d).
Logo, d tem de ser escolhido de modo que 2d < 2003 < 3d, o que implica que existem 4003 pares
possveis.
O primeiro jogador pode vencer se ele retira o par (1, 997). ssim, as possveis jogadas so:
(k, 997), para k = 2, 3, 4, . . . , 996, 998, . . . , 2003 (que so 2001 possibilidades), e (k, 1994),
para k = 1, 2, 3, . . . , 996, 998, 999, , 1993, 1995, 1996, . . . , 2003 (que so 2001 possibilidades).
Portanto, existe um nmero par de movimentos disponveis e o primeiro jogador vencer. Observe
que o segundo jogador s pode reduzir o nmero de movimentos disponveis se ele perder.

Exemplo-82:

Num hotel os quartos esto dispostos como um tabuleiro 2 8, estando cada um deles
ocupado por um hspede. Todos os hspedes esto se sentindo desconfortveis em seus
aposentos, de modo que cada um deles gostaria de passar para um quarto adjacente, horizontal
ou verticalmente. Naturalmente, eles podem fazer a mudana simultaneamente, de tal maneira
que cada quarto fique novamente ocupado por um hspede.
De quantas maneiras distintas podem ser feitos esses movimentos coletivos?

Soluo

(HMITMT - 2003) Imagine que os quartos so pintados alternadamente de preto e branco, no estilo de
um tabuleiro de xadrez, veja figura a seguir.

Pela figura, fcil ver que, cada hspede se movimenta de um quarto preto para um quarto adjacente
branco, e vice-versa. Se para um hspede, que inicialmente est num quarto preto, colocamos um
domin sobre o quarto antigo e o novo, obtemos uma pintura do tabuleiro do tipo 2 n, formada por
n domins, pois cada quarto preto usado uma vez e cada quarto branco usado uma vez. Aplicando
procedimento anlogo para cada hspede que inicialmente est num quarto branco e se movimenta
para um quarto preto, obtemos uma segunda pintura para o tabuleiro. Inversamente, fcil verificar
que, para qualquer par de quartos dessas pinturas, determina um padro de movimento. Tambm
fcil provar por induo que o nmero de pinturas do tipo domins de um tabuleiro de dimenses
2 n (n + 1)-simo nmero de Fibonacci.
Isto fcil de ver para os casos de n = 1, 2 e para um retngulo 2 n, onde os dois quadrados mais a
direita pertencem a um domin vertical ou deixam um retngulo de dimenso 2 (n 1) ser pintado
arbitrriamente, ou dois domins ocupam quartos adjacentes, deixando um retngulo de dimenso
72 Captulo 3. Problemas Diversos

2 (n 2) ser pintado livremente. Portanto, o nmero de pinturas possveis o nmero de Fibonacci.


Deste modo, o nmero de pinturas de um tabuleiro de dimenso 2 8 34, e o nmero possvel de
pares de tais pinturas igual a 342 = 1156, que a resposta do problema.

Exemplo-83:

Sem levar em considerao a ordem das parcelas, de quantas maneiras podemos expressar
2002 como soma de 3 inteiros positivos?
(Consideramos 2 + 1000 + 1000 e 1000 + 2 + 1000 como sendo a mesma forma de
expressar 2002 como soma de 3 inteiros positivos)

Soluo

(HMITMT - 2002) Sejam a, b e c trs nmeros positivos para os quais a + b + c = 2002. Para
facilitar o entendimento e evitar redundancia, vamos supor que a b c. Observe que, para cada
escolha de a e b, existe uma nica escolha para c, pois c = 2002 (a + b). Agora, vamos contar
todas as possibilidades para a e b considerando dois casos:
Caso 1 O nmero a par.
Caso 2 O nmero a mpar.
Suponha que o nmero a do terno procurado seja um nmero par. Neste caso, para cada valor de a,
teramos b = c = 2002a 2 , que um nmero inteiro. Com isso, teramos a + 2002a 2 + 2002a
2 = 2002,
2002a
satisfazendo ao problema. Observe que, para b = 2 a, segue que 2002 a 2a, que o
mesmo que a 667. Como a um nmero par, temos 2 a 666.
Para cada valor de a, existem ( 2002a 2 a + 1) valores para b. Ou seja, para cada valor de a,
2002a2a+2 3a
existem 2 = 1002 2 valores para b.
Agora, se a = 2, temos 1002 32 2 = 999 valores para b. Se a = 4, temos 2002 2 = 996
34
36
valores para b. Se a = 6, temos 2002 2 = 993 valores para b, e assim por diante at a = 666,
que nos d 2002 3666 2 = 3 valores para b. Para calcular a quantidade de todos os nmeros a, b, c
possveis, quando a um nmero par, somamos os nmeros obtidos anteriormente, que formam uma
progresso aritmtica. Isto , a quantidade de valores possveis para os ternos de nmero a, b, c, com
a par, satisfazendo ao problema igual a: 999 + 996 + 993 + ... + 3 = 999+3 2 333 = 501 333 =
166833.
Seja a um nmero mpar. Neste caso, para cada valor de a, existem [ (2002(a+1) 2 a + 1] =
[ (2002a12a+2
2 ] = [ 20003(a1)
2 ] = 1000 [ 3(a1)
2 ] possveis valores para o nmero b.
Como dado o nmero a, tem-se: b = 2002(a+1) 2 a, segue que 2002 (a + 1) a, que o mesmo
que a 667. Como a um nmero mpar positivo, temos 1 a 667.
Agora, se a = 1, temos que existem 1000 3(11) 2 = 1000 valores possveis para b. Se a = 3,
existem 1000 3(31) 2 = 997 valores possveis para b. Se a = 5, existem 1000 3(51) 2 = 994
3(6671)
valores possveis para b, e assim por diante at a = 667, que nos d 1000 2 = 1 valor
possvel para b. Para calcular a quantidade de todos os nmeros a, b, c possveis, quando a um
nmero mpar, somamos os nmeros obtidos anteriormente, que formam uma progresso aritmtica.
Isto , a quantidade de valores possveis para os ternos de nmero a, b, c, com a mpar, satisfazendo
ao problema igual a:
1000 + 997 + 994 + . . . + 1 = 1000+1 2 334 = 1001 167 = 167167.
Portanto, existem 166833 + 167167 = 334000 possveis arranjos dos nmeros inteiros positivos
a, b, c satisfazendo ao problema.
73

Exemplo-84:

Um tesouro est enterrado num tabuleiro de xadrez 8 8. Em cada uma das casas do tabuleiro
existe enterrado uma mensagem que indica o nmero mnimo de passos que se necessita para
chegar a casa do tesouro. Necessita-se de um passo para se mover de uma casa para a casa
vizinha, que uma que tem um lado comum com a casa de partida.
Determine o nmero mnimo de casas que se deve escavar para chegar com certeza ao tesouro.

Soluo

(TT - 2012) Se escolhemos uma casa qualquer para escavar, no h nenhuma garantia de que o
tesouro esteja l. Se a mensagem retirada diz que o tesouro est num outro quadrado, fazendo uma s
escavao, no possvel determinar a exata localizao dele. Vamos provar que, para encontrar o
tesouro, temos que escavar duas casas.
Numeramos as linhas do tabuleiro 8 8, de baixo para cima, de 1 a 8 e as colunas, da esquerda
para a direita, de 1 a 8. Assim, cada casa do tabuleiro perfeitamente identificada por um par de
nmeros (b, c), com 1 b, c 8, veja figura a seguir.

Linha 8 (8,1) (8,2) (8,3) (8,4) (8,5) (8,6) (8,7) (8,9)


Linha 6 (7,1) (7,2) (7,3) (7,4) (7,5) (7,6) (7,7) (7,8)
Linha 6 (6,1) (6,2) (6,3) (6,4) (6,5) (6,6) (6,7) (6,8)
Linha 5 (5,1) (5,2) (5,3) (5,4) (5,5) (5,6) (5,7) (5,8)
Linha 4 (4,1) (4,2) (4,3) (4,4) (4,5) (4,6) (4,7) (4,8)
Linha 3 (3,1) (3,2) (3,3) (3,4) (3,5) (3,6) (3,7) (3,8)
Linha 2 (2,1) (2,2) (2,3) (2,4) (2,5) (2,6) (2,7) (2,8)
Linha 1 (1,1) (1,2) (1,3) (1,4) (1,5) (1,6) (1,7) (1,8)
Coluna 1
Coluna 2
Coluna 3
Coluna 4
Coluna 5
Coluna 6
Coluna 7
Coluna 8

Escavamos as casas (1, 1) e (8, 1). Sejam a e b, respectivamente, os nmeros que encontramos aps
escavarmos as duas casas. Cada um desses nmeros representa o comprimento do caminho mnimo
que devemos fazer da casa escavada at o tesouro. Podemos imaginar, sem perda de generalidade, que
o caminho mnimo realiza primeiro todos os movimentos verticais necessrios em seguida e, aps isso,
os movimentos horizontais at atingir o tesouro.
Vamos supor que o tesouro esteja na casa (i, j) do tabuleiro. Os caminhos mnimos para se atingir o
tesouro, a partir das casas escavadas, so de dois tipos:
Tipo (I) - aquele que comea na casa (1, 1), vai at a casa (1, j) e da at a casa (i, j).
Tipo (II) - aquele que comea na casa (8, 1), vai at a casa (8, j) e da at a casa (i, j).
Deste modo, sejam V1 e H1 a quantidade de movimentos verticais e horizontais, respectivamente,
a partir da casa (1, 1) at chegar ao tesouro e V2 e H2 a quantidade de movimentos verticais e
horizontais
 a partir da casa (8, 1) at chegar ao tesouro. Desse modo, temos que:
V1 + H1 = a
V2 + H2 = b
Pelo que indicamos acima, os movimentos horizontais dos dois caminhos so os mesmos, i. e.
H1 = H2 , e a diferena | a b | igual a diferena entre os comprimentos dos movimentos verticais
dos dois caminhos. Alm disso, a soma dos comprimentos dos dois movimentos verticais igual a
V1 +V2 = 7.
Portanto, supondo a b, teremos um sistema de equaes lineares 2 2:
74 Captulo 3. Problemas Diversos

V1 V2 = a b
V1 +V2 = b
Este sistema tem soluo nica, o que determina exatamente a posio do tesouro.
Para ilustrar, considere o tabuleiro a seguir, onde em cada est escrito o nmero obtido com a escavao
e a identificao da casa onde se encontra o tesouro.

9 8 7 6 5 4 5 6
8 7 6 5 4 3 4 5
7 6 5 4 3 2 3 4
6 5 4 3 2 1 2 3
5 4 3 2 1 X 1 2
6 5 4 3 2 1 2 3
7 6 5 4 3 2 3 4
8 7 6 5 4 3 4 5

Quando escavamos a casa (1, 1) obtemos o nmero a = 8. Isto , o caminho mnimo de (1, 1) at
o tesouro tem comprimento 8. Quando escavamos a casa (8, 1) obtemos o nmero b = 9. Ou seja,
o caminho mnimo de (8, 1) at o tesouro tem comprimento 9.
Para se mover da casa (1, 1) at o tesouro, localizado neste exemplo na casa (4, 6), temos que fazer
um movimento vertical at a casa (5, 1) e de l fazemos um movimento horizontal at a casa (5, 6).
Alm disso, a diferena 9 8 = 1. Logo, temos que encontrar dois nmeros (entre 1 e 8) cuja soma
seja 7 e cuja diferena seja 1. Os nicos nmeros que satisfazem so 3 e 4. Portanto, de (1, 1)
temos que subir 3 casas e, de (8, 1), mover para baixo 4 casas. Como o comprimento do caminho
mnimo de (1, 1) at o tesouro 8, temos que andar 5 casas horizontalmente. Portanto, o tesouro
est na casa (4, 6).

Exemplo-85:

Num tabuleiro 8 8 comum, com as casas pintadas alternadamente de preto e branco, um


movimento permitido trocar de posio quaisquer duas linhas ou colunas.
Aplicando uma sequncia de tais movimentos, possvel obter um tabuleiro onde a metade es-
querda seja totalmente formada por casas pretas e a metade direita seja formada inteiramente
por casas brancas?

Soluo

(LMO - 1987) Observe que, em qualquer movimento permitido, no se altera o nmero de quadrados
pretos numa coluna do tabuleiro, que inicialmente quatro. Logo, nunca obteremos uma coluna
consistindo de oito quadrados pretos. Portanto, impossvel obter um tabuleiro onde a metade
esquerda seja totalmente formada por casas pretas e a metade direita seja formada inteiramente por
casas brancas.
75

Exemplo-86:

Um mgico tem cem cartes numerados de 1 a 100. Ele coloca os cartes em trs caixas,
uma vermelha, uma branca, uma azul, de tal modo que cada caixa contm no mnimo um
carto. Uma pessoa da plateia escolhe duas dessas caixas e retira cartes de cada uma delas.
Em seguida, revela a soma dos nmeros daqueles cartes. Dada esta informao, o mgico
localiza a caixa da qual nenhum carto foi retirado.
De quantas maneiras o mgico pode repartir os cartes nas caixas de modo que ele possa
sempre executar seu truque com xito?
(Duas maneiras de repartir os cartes so consideradas distintas se ao menos um carto
colocado em duas caixas diferentes)

Soluo

(OIM - 2000) A resposta 12.


Vamos chamar, respectivamente, de r, w, b as cores das caixas vermelha, branca, azul.
Vamos dizer que a cor do nmero i seja a da cor da caixa em que ele se encontra e todo nmero
considerado esteja no conjunto {1, 2, 3, . . . , 100}.
Para que o truque funcione, sempre que x + y = z + t e os cartes de nmeros x e y estejam em
casas distintas, ento ou z e t esto nestas mesmas caixas ou ambos esto na caixa restante. Isso
significa dizaer que, nenhum par de nmero de cores distintas tem a mesma soma que outros dois em
caixas distintas.
Consideramos dois casos:
Caso 1: Suponhamos que existe uma caixa tal que os cartes numerados com i, i + 1, i + 2 so de cores
distintas, digamos vermelho (r), branco (w), azul (b), respectivamente.
Questo: Em que caixa est o carto de nmero i + 3?
Est na caixa vermelha. De fato, como |{z} i + (i + 3) = (i + 1) + (i + 2), para que o truque funcione,
| {z } | {z } | {z }
r ? w b
(i + 3) tem de ser vermelho (r).
De modo anlogo, como (i + 4) + (i + 1) = (i + 3) + (i + 2), para que o truque funcione, segue que
| {z } | {z } | {z } | {z }
? w r b
(i + 4) branco (w). Do mesmo modo, como (i 1) + (i + 2) = (i) + (i + 1), segue que (i 1)
| {z } | {z } |{z} | {z }
? b r w
branco (w)
Com isso, conclumos que trs cores vizinha distintas determinam a cor do prximo nmero. Alm
disso, este processo se repete: rwb seguido por r, w, b e assim por diante. Este processo tambm
funciona inversamente: rwb precedido por b, e assim por diante.
Logo, suficiente definir as cores dos cartes numerados com 1, 2, 3, que pode ser feito de 6 modos
distintos. Todos esses arranjos so tais que as somas dos nmeros vermelhos e azuis, dos nmeros
brancos e azuis, e dos nmeros vermelhos e azuis, so restos distintos mdulo 3. Isto , a primeira caixa
comtm os carte de nmeros 1, 4, 7, , 100, a segunda caixa contm os de nmeros 2, 5, 8, , 98 e
a terceira contm 3, 6, 9, , 99
Caso 2: Suponhamos que no existem trs cartes com nmeros consecutivos em caixas distintas.
Sem perda de generalidade, suponhamos que o nmero 1 seja vermelho (r).
Agora, seja i o maior nmero que no vermelho. Vamos supor que i seja branco. Seja k o menor
nmero azul. Como, por hiptese, no existe trs nmeros consecutivos de cores distintas, rwb, temos
necessariamente que (i + 1) < k.
Suponhamos que k < 100. Como |{z} i + |{z} k = (i 1) + (k + 1), segue que (k + 1) vermelho (r).
| {z } | {z }
w b r ?
i + (k + 1) = (i + 1) + |{z}
No entanto, como |{z} k , segue que (i + 1) deve ser azul b, o que contradiz o
| {z } | {z }
w r ? b
fato de que k o menor nmero azul. Logo, k = 100.
Do mesmo modo, como (i 1) + |{z}
100 = |{z}
i + |{z}
99 , segue que 99 branco.
| {z }
r b w ?
76 Captulo 3. Problemas Diversos

Se t um nmero inteiro, com t > 1 e o carto numerado com t esteja na caixa vermelha (r), como
99 = (t 1) + |{z}
t + |{z}
|{z} 100 , temos que (t 1) azul (b), segue que o menor nmero azul 100.
| {z }
r w ? b
Portanto, a pintura dos nmeros feita do seguinte modo: rww wwb.
Deste modo, se a soma maior do que ou igual a 100, ento a caixa restante azul; se a soma
101, a caixa restante branca; e se a soma maior do que 101, a caixa restante vermelha. Ou
seja, uma caixa contm o carto com nmero 1, uma outra caixa contm os cartes com os nmeros
2, 3, 4, , 99 e a outra caixa contm somente o carto com nmero 100. O nmero de tais arranjos
6.
Portanto, a resposta do problema 6 + 6 = 12.

Exemplo-87:

Provar ou d um contra-exemplo: Dado um conjunto finito de pontos no plano com


coordenadas inteiras, possvel pintar de vermelho alguns destes pontos e os restantes de
branco de tal forma que para qualquer reta L paralela a um dos eixos coordenados, o
nmero de pontos vermelhos e o nmero de pontos brancos sobre L diferem por no mximo 1.

Soluo

(OIM - 1986) A resposta sim.


Sejam P1 , P2 , P3 , , Pn pontos sobre uma reta r paralela a um eixo, contados da esquerda para a
direita e de cima para baixo. Desenhamos segmentos juntando P1 a P2 , P3 a P4 , . De um modo
geral, traamos um segmento juntando os pontos P2i1 a P2i . Feito isso para toda dessas retas
r, obtemos uma algumas linhas poligonais. Se uma dessas poligonais fechada, significa que ela
forma um poligono com um nmero par de vrtices. Agora, pintamos os vrtices dessa poligonal
alternadamente de vermelho e branco. Um ponto que no esteja sobre a poligonal pode ser pintado de
forma arbitrria. A pintura obtida satisfaz ao problema.
Exemplo-88:

Dadas 32 pedras de pesos distintos, prove que 35 pesagens num balana de dois pratos so
suficiente para determinar quais so as duas pedras mais pesadas.

Soluo

(LMO - 1989) Inicialmente, dividimos as 32 pedras em 16 pares e, fazendo 16 pesagens, separamos


o conjunto das 16 pedras mais pesadas. Agora, dividimos as 16 pedras mais pesadas em pares
e, com 8 pesagens, determinamos as 8 mais pesadas dentre as 16 mais pesadas. Repetindo o
processo, dividimos as 8 pedras mais pesadas em pares e, com quatro pesagens, encontramos a
4 mais pesadas. Repetindo o processo, podemos determinar a mais pesada das pedras depois de
16 + 8 + 4 + 2 + 1 = 31 pesagens.
Durante as pesagens, a segunda pedra com mais peso s poderia ser eliminada pela pedra mais pesada.
Nesse momento, voc conhece a pedra mais pesada e as 6 mais pesadas. Agora, fique s com as 5
mais pesadas. Extraia as 5 pedras que foram pesadas contra a pedra mais pesada durante as pesagens.
Nesse momento, fcil de encontrar a mais pesado dessas 5 pedras em apenas 4 pesagens sucessivas,
fazendo a pesagem duas a duas e eliminando a mais leve.

Exemplo-89:

Um polgono convexo possui 2n lados. Prove que o polgono contm no mnimo n diagonais
que no so paralelas a qualquer um de seus lados.
77

Soluo

(SAMO - 1996)Como o polgono possui 2n lados, ele possui 2n.(2n3) 2 = n(2n 3) diagonais.
Agora, observe que, para cada lado, existem no mximo (n 2) diagonais paralelas a ele. Deste
modo, existem no mximo 2n(n 2) diagonais paralelas a algum lado. Portanto, existem no mnimo
n(2n 3) 2n(n 2) = n diagonais que no so paralelas a qualquer lado.

Exemplo-90:

Na dcada de 1960, tinha um programa semanal de televiso muito popular nos Estados
Unidos chamado "Lets Make a Deal"("Vamos fazer um acordo"). A cada semana, em um
determinado momento no programa, o apresentador, Monty Hall, apresentava ao competidor
trs portas fechadas.

Atrs de uma das portas, havia um prmio substancial (vamos supor que fosse um carro)
e atrs das outras, no havia nada substancial (vamos imaginar que tivesse um bode). O
apresentador, Monty Hall, pedia que o participante escolhesse uma das porta. Claramente, a
chance de o competidor escolher a porta com o prmio era de 1 em 3. Para causar impacto
aos telespectadores, em vez de simplesmente abrir a porta escolhida para revelar o que est
por trs, o apresentador, Monty Hall, abria uma das duas portas que o competidor no tinha
escolhido, revelando que ela no escondia o prmio (e sim um bode).

claro que o apresentador sabia onde o prmio estava, por isso ele podia fazer isso. Ele ento
oferecia ao competidor a oportunidade de mudar sua escolha para a outra porta que, naquela
momento, permanecia fechada.
Qual a estratgia mais lgica, ficar com a porta escolhida inicialmente ou mudar de porta?
Com qual das duas portas ainda fechadas o concorrente tem mais probabilidades de ganhar?
Por qu?

Soluo

Existem trs portas: A, B e C.


Quando o competidor escolheu uma delas, digamos a porta A, a chance de que ela seja a premiada
de 13 . Como consequncia, a probabilidade de que tenha errado, ou em outras palavras, de que o
prmio esteja nas outras duas portas B ou C de 23 .
Pode-se comprovar isso somando a probabilidade de cada uma das outras portas ou simplesmente
sabendo que a probabilidade de que haja um prmio sempre 1.
O importante ter em mente que a chance de o prmio estar nas outras portas que ele no escolheu
de 32 .
Entendendo isso, basta ver que o apresentador abrir sem erro uma dessas outras duas portas que
contm um bode, digamos que seja a porta B. Ao fazer isso, ele est dando uma informao valiosa:
se o prmio estava nas outras portas que no escolheu (B ou C), ento agora ele s pode estar na
porta que o competidor no escolheu e no foi aberta, ou seja, a porta C. Ou seja, se o competidor
errou ao escolher uma porta - e as chances disto so de 2/3 - ento ao abrir uma das outras portas
no-premiadas o apresentador est lhe dizendo onde est o prmio. Toda vez que o competidor tiver
escolhido inicialmente uma porta errada, ao trocar de porta ir com certeza ganhar. Como as chances
de que tenha errado em sua escolha inicial so de 23 , se trocar suas chances de ganhar sero de 23 - e
78 Captulo 3. Problemas Diversos

por conseguinte a chance de que ganhe se no trocar de porta de apenas 13 . assim mais vantajoso
trocar de porta.
Observao
Suponha que, em vez de 3 houvessem 1000 portas, numeradas de 1 a 1000, e o competidor
escolheu a porta de nmero 1000.

Todos as portas contm um bode, exceto uma, aquela que tem o carro.

Quando o competidor escolhe uma porta, em seguida, Monty Hall abre todas as outras portas, exceto
uma... e d a oportunidade para o competidor mudar a escolha para a outra porta. Se fosse voc,
mudaria? possvel um pensamento do tipo "Escolhi a porta correta no meu primeiro palpite."Mas,
1
qual a probabilidade de que isso fosse verdade? Resposta: 1000 ... H uma chance de 999 de que o
carro no est atrs da porta que o competidor escolheu. E se o carro no est atrs da porta que o
competidor escolheu, ele deve estar por trs da ltima porta que Monty deixou sem abrir. Em outras
palavras, Monty ajudou, deixando uma porta para que o competidor mude, que tem uma chance de
999
999 de ter o carro por trs dela. Ento nesse caso, se voc mudar, voc teria uma chance de 1000 de
ganhar o carro. Portanto, melhor mudar.

Exemplo-91:

Dois jogadores, A e B, disputam um jogo em que jogam alternadamente. Uma jogada de A


escolher um ponto do plano, que no esteja pintado, e pint-lo de vermelho. Uma jogada de B
escolher 10 pontos do plano, que no estejam pintados, e pint-los de verde. O jogador A
vence se existem trs pontos vermelhos que sejam vrtices de um tringulo equiltero.
O jogador B pode impedir do jogador A de vencer a partida?

Soluo

A resposta no.
O primeiro jogador, A, pode sempre vencer, no importa como o jogador B jogue. Para isso, em suas
n primeiras jogadas ele escolhe pintar de vermelho pontos sobre uma mesma reta. Agora, observe que
para cada par destes n pontos existem dois pontos, um de cada lada da reta, que o segundo jogador,
B, tem de pintar de verde, para evitar que o jogador A vena.
O nmeros de tais pontos iguala 2 n2 . Por outro lado, o jogador B pode pintar no mximo 10n
pontos de verde. Agora, 2 n2 > 10n, que o mesmo que n(n 1) > 10n. Ou seja, n 1 > 10.
Portanto, o primeiro jogador vence no seu 12o movimento.

Exemplo-92:

Ana e Clia disputam o jogo seguinte, em que jogam alternadamente. Ana deve pintar, ou
de azul ou de vermelho, cada ponto que esteja sobre um dado crculo. Clia deve escolher
trs dos pontos pintados de modo que eles determinem um tringulo cujas medidas de seus
ngulos internos sejam 30, 50 e 100 graus, respectivamente. Clia vence se esse tringulo
possui os vrtices de mesma cor. Ana vence se Clia no consegue fazer a escolha.
Quem vence: Ana ou Clia?

Soluo
79

Ana vence.
Basta ela marcar 6 pontos sobre o crculo, de modo que este fique dividido em seis arcos de mesmo
comprimento. Agora ela pinta alternadamente de azul e vermelho cada um dos arcos, de forma
consecutiva, tendo o cuidado de pintar em cada arco o ponto inicial da cor do arco, mas deixar o
ponto final com a cor do prximo arco. Quando Clia quiser escolher os vrtices de seu tringulo, os
extremos do lado oposto ao ngulo de 600 , ela deve determinar um arco de 600 sobre o crculo. Pela
pintura feita, estes extremos devem necessariamente possuir cores distintas.

Exemplo-93:

Numa caixa, temos 25 fichas iguais, numeradas de 1 at 25. Voc pode retirar da caixa
uma ficha por vez, sem reposio, e pode continuar a retirar fichas at que o produto de dois
nmeros de duas fichas retiradas seja um quadrado perfeito.
Qual o nmero mnimo de fichas que voc deve retirar para ter certeza de obter um par de
fichas cujo produto um quadrado perfeito?

Soluo

Seja S = {1, 2, 3, . . . , 24, 25}. Partimos o conjunto S em subconjuntos disjuntos com as seguintes
duas propriedades:
(i) o produto de quaisquer dois elementos de um subconjunto um quadrado perfeito; (ii) o produto de
quaisquer dois elementos escolhidos em subconjuntos distintos no um quadrado perfeito. Assim,
escrevemos:

S = {1, 4, 9, 16, 25} {2, 8, 18} {3, 12} {5, 20} {6, 24} {7} {10}

{11} {13} {14} {15} {17} {19} {21} {22} {23}


.
Ou seja, partimos o conjunto S em 16 subconjuntos disjuntos. Se escolhermos uma ficha de cada
um dos subconjuntos no teremos obtido um par de fichas satisfazendo ao problema. Mas, com mais
uma ficha vamos obter um par de fichas cujo produto um quadrado perfeito. Portanto, a resposta
16 + 1 = 17.

Exemplo-94:

Dois jogadores, A e B, disputam um jogo, em que jogam alternadamente. Inicialmente,


escreve-se no quadro-negro o nmero natural 2. Uma jogada consiste em somar ao ltimo
nmero, n, no quadro-negro um divisor prprio de n. Quem atingir um nmero maior do
que ou igual 2010 vence. O jogador A comea o jogo.
Quem vence, A ou B? Qual a estratgia para vencer?

Soluo

O jogador A possui uma estratgia vencedora.


A estratgia do jogador A consiste em jogar deixando para seu adversrio sempre um nmero mpar.
Observe que um divisor prprio de um nmero mpar no mximo um tero daquele nmero, pois o
menor divisor deste nmero 3. Deste modo, o jogador B s pode somar no mximo 13 do ltimo
nmero escrito no quadro-negro.
Toda vez que o jogador B vai jogar, encontra um nmero mpar e soma a ele um divisor dele, que
mpar tambm, resultando sempre num nmero par. O jogador A por sua vez soma sempre um
divisor de um nmero par. Assim, ele pode jogar a metade deste nmero e prossegue assim at obter
pela primeira vez um nmero maior do que ou igual a 1340. Uma vez obtido esse nmero x, o
jogador A soma a ele sua metade, atingindo ento um nmero maior do que 2010. (Observe que:
80 Captulo 3. Problemas Diversos
x+x
2 2010 x 1340).

Exemplo-95:

Dois jogadores, A e B, disputam o seguinte jogo, em que jogam alternadamente.


Escrevem-se no quadro-negro uma sequncia de 49 nmeros inteiros consecutivos quaisquer.
Uma jogada consiste em apagar um desses nmeros. No final, restam somente dois nmeros:
a e b. O jogador A ganha se MDC(a, b) = 1 e B ganha se MDC(a, b) > 1. O jogador A
comea o jogo.
(a) Quem ganha: A ou B? Qual a estratgia para vencer?
(b) Se, em vez de 49 nmeros inteiros consecutivos fossem 50, quem ganha: A ou B?
Qual a estratgia para vencer?

Soluo

(a) O jogador A ganha.


Como a quantidade de inteiros mpar, o jogador divide os 49 primeiros nmeros em pares de inteiros
sucessivos, deixando o nmero final sozinho. Ele inicia apagando o nmero que ficou sozinho. Se o
jogador B apaga qualquer nmero de um par, ento A apaga o outro. O que sobra no final um par
(n, n + 1), com MDC(n, n + 1) = 1.
(b) O jogador B ganha.
A estratgia do jogador B apagar todos os nmeros mpares, deixando dois nmeros mpares
mltiplos de 3. Por sua vez, o jogador A sempre apaga um nmero par. Antes da penltima jogada
restam quatro nmeros no quadro-negro: dois nmeros pares, p e q, e dois nmeros mpares, i e
j. Se A apagar um nmero mpar, ento B apaga o outro nmero mpar, restando os dois nmeros
pares. Se A apaga um dos nmeros pares, ento B apaga o outro nmero par, restando i e j, com
MDC(i, j) 3.

Exemplo-96:

Todo membro de uma seqncia de nmeros, a partir do segundo, igual a soma do termo
precedente com a soma de seus dgitos. O primeiro nmero da seqncia 1.
Diga, justificando, se o nmero 2010 pertence seqncia.

Soluo

A resposta no.
Seja a1 , a2 , a3 , a4 , . . . a sequncia de nmeros. Observe que:

a1 = 1,

a2 = 2 = 3.0 + 2,
a3 = 4 = 3.1 + 1,
a4 = 8 = 3.2 + 2,
a5 = 16 = 3.5 + 1,
a6 = 23 = 3.7 + 2,
a7 = 28 = 3.9 + 1,
...............
Para cada inteiro m, seja S(m) a soma dos dgitos de m. Sabe-se que, na diviso por 3, os nmeros
m e S(m) deixam o mesmo resto. Observando os nmeros da seqncia encontrados acima, temos
que os restos da diviso por 3 dos termos da seqncia so alternadamente 1 e 2. De fato, se
81

an = 3k + 1, com k um nmero inteiro, ento an+1 = an + S(an ) = (3k + 1) + (3q + 1) = 3 j + 2,


onde q e j so nmeros inteiros.
Se an = 3k + 2, com k um nmro inteiro, ento an+1 = an + S(an) = (3k + 2) + (3q + 2) = 3 j + 1,
onde q e j so nmeros inteiros.
Deste modo, de fato o resto da diviso dos termos da sequncia por 3 ou 1 ou 2. Mas, 2010
deixa resto zero na diviso por 3. Portanto, 2010 no aparece na sequncia.

Exemplo-97:

No aniversrio de Maria, Jos quer dar-lhe um presente e, para isso, prope o jogo se-
guinte, no qual eles jogam alternadamente. Ele escreve no quadro-negro os nmeros inteiros
0, 1, 2, 3, 4, . . . , 255, 256. Maria comea o jogo. Ela escolhe 27 dos nmeros escritos e
apaga-os. Jos escolhe 26 nmeros dentre o que sobraram e apaga-os. Em seguida, Maria
escolhe 25 dentre os nmeros restantes a apaga-os. E eles prosseguem assim at que Jos
escolhe 20 = 1 nmero e apaga-o. Como so apagados

27 + 26 + 25 + 24 + . . . + 21 + 20 = 28 1 = 255

nmeros, restam no quadro-negro dois nmeros, a e b. Jos paga para Maria uma quantia,
em reais, correspondente ao valor |a b|.
Se os jogadores fazem sempre suas melhores jogadas, qual o maior valor que Maria tem
certeza que ganha?

Soluo

(CRUX) Maria ganha no mximo 24 ou 216 reais.


A estratgia de Maria apagar sempre os nmeros que esto nas posies pares. Com isso, ela aumenta
a distncia entre eles, aumentando o valor de |a b|. Depois das jogadas de nmeros 1, 2, 3 e 4,
a distncia entre os nmeros vizinhos, que esto ainda escritos no quadro-negro, 2, 4, 6, 8 e 16,
respectivamente, o que garante a Maria receber o mximo possvel, independente de como Jos joga.

Exemplo-98:

Daniel tem uma balana de dois pratos e dez pedras cujos pesos so todos nmeros inteiros
entre 1 at 10, inclusive. Ele quer colocar algumas pedras na balana de tal forma que os
pratos fiquem equilibrados.
(a) Determine qual o maior e o menor nmero de pedras que pode colocar. Justifique por que
o maior e mostre um exemplo.
(b) Determine a maior quantidade de pedras que se pode colocar se em cada prato se deve
haver a mesma quantidade de pedras.

Soluo

(RP) (a) impossvel colocar 10 pedras, pois 1 + 2 + 3 + 4 + 5 + 6 + 7 + 8 + 9 + 10 = 55, que


um nmero mpar e no pode ser escrito como soma de dois inteiros iguais. Nove pedras podem
ser usadas, basta retirar uma pedra cujo peso um nmero mpar. Por exemplo, se retirarmos a
pedra 1, podemos colocar num dos pratos as pedras 8 + 9 + 10 = 27 e no outro prato as pedras
2 + 3 + 4 + 5 + 6 + 7 = 27.
(b) O mximo possvel de pedras que podemos usar 8. Por exemplo, descartamos as pedras 9 e 10
e usamos 1 + 4 + 5 + 8 = 18 num dos pratos e 2 + 3 + 6 + 7 = 18 no outro prato.
82 Captulo 3. Problemas Diversos

Exemplo-99:

Um motorista planeja uma viagem que vai passar por um trecho de 800 km no deserto,
que no possui postos para abastecer. Ele sabe que seu carro s pode armazenar 50 litros
de gasolina e tem um rendimento de 10 km por litro. O motorista pode deixar gasolina
armazenada em barris, no costado da rodovia, em pontos distintos dessa regio desrtica. Por
exemplo, com o tanque cheio com 50 litros de gasolina ele pode percorrer 100 km, deixar
armazenado 30 litros no ponto que chegou e voltar ao ponto de partida para reabastecer o
tanque de gasolina. O motorista decide realizar a viagem e chega ao primeiro posto antes do
deserto com tanque vazio.
Pode o motorista atravessar o deserto se neste primeiro posto de abastecimento antes do
deserto a oferta total de gasolina de 140 litros? E se fosse 180 litros?

Soluo

(OMP) Com a oferta de 140 litros no possvel o motorista atravessar o deserto.


Para atravessar o deserto se necessita armazenar gasolina no quilmetro 300 ou mais adiante, caso
contrrio, se percorreria mais de 500 km sem abastecer, o que impossvel, pois a autonomia do carro
de no mximo 500 km. Chamemos este ponto de depsito de combustvel de P. Para armazenar
gasolina em P o motorista tem de percorrer pelo menos duas vezes a distncia do primeiro posto ao
ponto P, que pelo menos 2 300 km = 600 km. Como a distncia a ser percorrida de 800 km,
vemos que impossvel atravessar o deserto com uma oferta menor do que 140 litros. Por outro lado,
no se pode atravessar o deserto com uma oferta exatamente igual a 140 litros, simplesmente porque
impossvel armazenar gasolina em P e voltar com esta quantidade.

Com 180 litros possvel atravessar o deserto seguindo o seguinte roteiro:


(a) O motorista faz trs viagens para armazenar gasolina no ponto P, situado a 50 km da partida,
armazenando 40 litros em cada uma das viagens;
(b) Aps estas trs viagens, abastece o tanque no primeiro posto com 30 litros, volta ao ponto P,
chegando com 25 litros;
(c) Do ponto P, avana duas vezes a um ponto Q a 100 km dele, armazenando ali 30 litros em
cada viagem;
(d) De volta ao ponto P abastece com 45 litros que esto armazenados ali e segue at o ponto Q
com 35 litros de gasolina;
(e) A partir de Q, chegando num ponto R, armazenando ali 20 litros de gasolina e volta ao ponto
Q;
(f) Abastece os 45 litros de gasolina restantes em Q, avana at R, chegando com 30 litros,
abastece ali com os 20 litros existentes no local e completa a viagem.

Exemplo-100:

Qual o valor mximo que se pode obter ao dividir um nmero natural de trs dgitos pela
soma de seus dgitos?
Justifique porque no se pode obter um valor maior.

Soluo

(Olimpada Matemtica Rioplatense) Todo nmero natural de trs dgitos da forma

a 100 + b 10 + c, com a, b, c {0, 1, 2, 3, 4, 5, 6, 7, 8, 9} e a 6= 0.

Vamos provar que


a 100 + b 10 + c
100.
a+b+c
83

De fato, basta observar que a 100 + b 10 + c a 100 + b 100 + c 100 = 100(a + b + c), o que
implica a100+b10+c
a+b+c 100.
Agora, observe que o valor 100 atingido quando a = 1, b = c = 0. Ou seja, quando o nmero
igual a 100:
100
= 100.
1+0+0

Exemplo-101:

O quadrado da figura a seguir, que est dividido em quadradinhos de dimenso 2 2, foi


construdo com palitos de fsforos de comprimento 2 cm.

Encontrar o tamanho do maior quadrado dividido em quadradinhos 2 2 que podemos


construir com 1999 palitos de fsforos. Quantos palitos sobraram? Quantos palitos tem em
cada lado deste quadrado?

Soluo

(OMA - 1999) Observe que, para construir um quadrado onde num lado usamos n palitos de fsforos,
dividido em quadradinhos 2 2, precisamos utilizar n(n + 1) palitos de fsforos na horizontal e
n(n + 1) palitos de fsforos na vertical. Isto , para construir um quadrado como se quer, usamos
n(n + 1) + n(n + 1) = 2n(n + 1) palitos de fsforos.
O que queremos encontrar o maior inteiro n para o qual

2n(n + 1) 1999.

Basta observar que, como

2 31 32 = 1984 < 1999 < 2 32 33 = 2112,

o maior valor possvel para n = 31. Neste caso, se usa 1994 palitos de fsforos, sobrando
1999 1984 = 15 palitos. O lado do quadrado que se constri utilizando-se os 1984 palitos
possui 2 31 = 62 palitos de fsforos.
4. ABREVIATURAS

4.1 Abreviaturas
AMO - Australian Mathematical Olympiad
AUMO - All Union Mathematical (URRS)
BW - Baltic Way Mathematical Contest
CMO - Canadian Mathematical Olympiad
CRUX - Crux Mathematicorum (Revista de resoluo de problemas - Canad)
HMITMT - Havard-MIT Mathematics Tournaments
IMO - International Mathematical Olympiad
KMO - Kiev Math Olympiad
LMO - Lenigrad Mathematical Olympiad
OMA - Olimpiada Matemtica Argentina
OMM - Olimpiada Mexicana de Matemtica
OMP - Olimpiada de Matemtica do Peru
RP - Olimpiada Rioplatense de Matemtica
SAMO - South African Mathematics Olympiad
TT - Tournament of the Tows
5. Bibliografia

ANDREESCU, Titu; SAVCHEV, Svetoslav - Mathematical Miniatures. The Mathematical


Association of America. Anneli Lax New Mathematical Library, Volume #43. Wasington . 2003.

ANGEL, D - http://math.stackexchange.com/questions/731193/how-to-win-this-game

KISACANIN, BRANISLAV - Mathematical Problems and Proof - Combinatorial, Number


Theory, and Geometry. Kluwer Academic Publishers. New York. 2002

BRIGGS, William - Ants, bikes & clocks - Problems solving for undergraduates. Society for
Industrial and Applied Mathematics - Siam. Philadelphia. 2005

EWING, J. - Paul Halmos: In his own words. Notice of AMS. Vol. 54 . Number 9. October 2007.

GUZMN, O. M. - Aventuras Matemticas. Gradiva. Lisboa. 1990.

CRUX MATHEMATICORUM. Canadian Mathematical Society.

MARTN, J. E. - Resolucion de Problemas Matemticos (Vol 3). Centro de Professores y Recur-


sos. Salamanca. 1999. (Em PDF)

POSAMENTIER, A.S.; WOLFGANG, S - The Art of Problem Solving. Corwin Press. Thousand
Oaks. 1996.

POSAMENTIER, A.S. - The Art of Solving Problems. Vienna 2-06 (Em PPT)

POSAMENTIER, A.S. - Problem Solving in Mathematics - Grade 3-6 Powerful Strategies to


Deeper. Corwin Press. Thousand Oaks. 2009.

ZEITZ, P. - The Art and Craft of Problem Solving. Second Edition. Wiley. Danvers. 2007

You might also like